Какие мышцы лучше тренировать вместе: Какие части мышц нужно качать вместе. Какие мышцы нужно тренировать вместе

Содержание

Какие части мышц нужно качать вместе. Какие мышцы нужно тренировать вместе

Один из самых распространенных вопросов перед проведением тренировки – что с чем тренировать, какие группы мышц прорабатывать вместе и как часто это делать?

Существует множество способов составить тренировочный план. Подберите наилучший вариант в зависимости от вашего опыта и целей. Если один тип тренинга не дает необходимой нагрузки, пробуйте выстроить программу иным способом, который будет учитывать вашу индивидуальность.

Для набора мышечной массы

Согласно рекомендациям спортивной медицины силовой тренинг следует проводить как минимум два раза в неделю. За это время вы разработаете основные группы мышц: груди, спины, плеч, бицепсов, трицепсов, пресса, передней и задней мышц бедра.

Помните, что необходимо 48 часов отдыха для восстановления любых мышц. Не забывайте, что для набора массы требуется уделять внимание качественному белковому питанию и соответствующей калорийности рациона.

Что с чем тренировать в тренажерном зале для набора массы? Не следует нагружать большое количество мышц одновременно, однако задействованные мышцы должны поработать на максимум, по возможности не затрагивая другую большую группу мышц.

В этом случае организм будет прицельно восстанавливать определенные участки, не растрачивая строительный материал на приведение всего тела в норму.

Тренировка противоположных групп мышц – один из наиболее распространенных видов силового тренинга. При этом в одной сессии делают упражнения на мышцы-антагонисты, которые обеспечивают движение суставов в противоположных направлениях.

Проще говоря, в один день вы работаете над передней и задней стороной конкретной области тела.

Поскольку потребуется отдых для нагруженных мышц, удобно на следующий день провести тренировку на другую часть тела.

Преимущества этого метода:

  • Целевая группа мышц работает на максимум. Вы не сможете добиться такого же эффекта при тренировке всего тела сразу.
    Каждый раз фокусируясь на отдельной области, вы быстрее получите результат.
  • Исследования утверждают, что вы активнее нагружаете мышечные волокна, когда в одной и той же сессии прорабатываете или хотя бы растягиваете мышцы-антагонисты. Это означает, что жим на мышцы груди будет эффективнее после растяжки средней части спины.
  • Тренировка мышц-антагонистов гарантирует, что вы проработаете обе стороны тела равномерно, сохраняя баланс развития фигуры.

К недостаткам такого вида тренировки можно отнести то, что вы проработаете каждую группу мышц только раз в неделю. Примерное расписание для четырех тренировок в неделю:

  • Понедельник: грудь и спина.
  • Вторник: передняя и задняя мышцы бедра, пресс.
  • Четверг: бицепсы, трицепсы и плечи.
  • Суббота: грудь и спина.

В следующий понедельник выполняйте упражнения на мышцы бедра и пресса и продолжайте ротацию далее.

Еще один вариант прицельной нагрузки мышц – разделить большие группы мышц по дням. Таким образом, в один день вы тренируете грудь и трицепсы, на другой день – спину и бицепсы, в следующую тренировку – ноги и плечи.

Плечи можно прорабатывать вместе с грудными мышцами. Этот способ тренинга также удобен при плотном тренировочном плане.

Имейте в виду, что для повышения показателей и разгрузки нервной системы тренировочную стратегию необходимо периодически изменять. Например, ставить упражнения на бицепс вместе с грудными мышцами, а спину вместе с трицепсами. Это разнообразит тренировки, снимет психологическую усталость и придаст импульс дальнейшему набору мышечной массы.

Для похудения

Если ваша цель – похудение, тренировка должна отнимать максимум энергии и эффективно сжигать калории.

Что с чем тренировать в один день для похудения? Хорошо подойдет функциональный тренинг, в котором за одну сессию прорабатываются практически все группы мышц.

В этом случае необходимо делать большое количество подходов (от 8) с максимальными весами, но малым числом повторений за один подход.

В каждую из тренировок включают все 3 упражнения из пауэрлифтинга, однако в день проработки грудных мышц проводится облегченная тренировка на спину и ноги.

Сначала проработайте большие мышечные группы: ноги, грудь и спину. Например, становая тяга и тяга в наклоне заставляют работать сразу несколько групп мышц. Затем перейдите к проработке мелких групп: трицепсов, бицепсов и плеч. Делайте по 2 упражнения на каждую группу за тренировку, чтобы избежать перетренированности.

По мере развития силы и выносливости, вы сможете разнообразить и усилить тренинг (добавить упражнений или увеличить веса).

Новички часто полагают, если каждый день качать пресс – это быстро избавит их от жира на животе и прорисует заветные «кубики». На самом деле, мышцы живота работают по тому же принципу, как и любая другая группа мышц. Поэтому имеет смысл давать им хотя бы день отдыха между тренировочными днями.

Проработку мышц пресса часто соединяют с упражнениями на спину или ноги.

Привет покорителям железных рубежей! В одном из последних сюжетов мы говорили о вашей мечте и постановке цели для того, чтоб мотивировать себя на ее достижение. Однако, как говориться, мечты – это планы в уме, а планы – это мечты на бумаге.. Тема нашего сегодняшнего сюжета – это, как вы уже догадались, тренировочные планы, или программы. Я достаточно педантичный человек и верю, что даже самый дурацкий замысел можно выполнить мастерски, если долго и систематически следовать плану. Кстати, тут вы сразу же услышали и первый совет — для отдачи от любого плана нужно время и последовательность. Часто новички прыгают с одного тренировочного сплита на другой, просто не давая своему телу прочувствовать преимущества конкретной тренировочной программы. Если вы не хотите постоянно топтаться на месте, то будьте готовы выбрать план следовать ему многие месяцы, иначе вы просто не поймете, эффективен ли он вообще для вас или нет. Ок. Как же составляют тренировочную программу персональные тренеры?

Персональный Тренер и Тренировочная Программа.

Каждый человек, когда начинает заниматься, хочет верить в некие секретные методы и программы тренировок, так же, как волшебные протеины и прочие таблетки. Это так называемый синдром «волшебной палочки». С этой целью новички идут к персональным тренерам и просят написать им «волшебную» Персональную программу, от которой их мышцы просто разорвет и они станут похожи на Арнольда Шварценеггера через пол года. К счастью, чудеса бывают только в сказках, а результат всегда требует тяжелой работы над собой. Так что готовьтесь потеть, друзья.

Чаще всего Персональный Тренер всем новичкам пишет одну и ту же программу с небольшими вариациями на свой вкус. К примеру, если подойдет девушка или худой подросток – даст больше повторений в каждом подходе и выкинет некоторые опасные упражнения из тренировочного сплита. В целом же, если у тренера сложились определенные представления о тренинге, он будет всем своим клиентам давать практически одно и то же. Плохо ли это? Чаще всего, если тренер не полный дуб, то это не так уж и плохо. Потому что идеально подходящую персонально для вас программу составить с первого раза НЕ ВОЗМОЖНО! Это все равно, что во время боевых действий из миномета без пристрелки и коррекции сразу попасть в далекую цель. Наводчик обычно делает первый выстрел, смотрит на результат, делает коррекцию, второй выстрел, и вот уже с третьего он, зная все особенности, может рассчитывать на прямое попадание. Так же и в составлении тренировочной программы. Пока мы не знаем, как наше тело будет реагировать на ту или иную нагрузку, мы не можем выбрать самый лучший способ прогрессии этой нагрузки.

Главное, чтоб тренер не начал писать уж полную отсебятину. Ну, к примеру, заставил бы вас делать 20-30 повторений или отдыхать 3 минуты, в то время как ваша цель – набор мышечной массы. Короче, если коротко, основная задача при составлении первой тренировочной программы – это выбрать примерно верное направление. И в последствии это направление корректировать для лучшей точности и результативности.

Поэтому любая программа лучше, чем вообще ее отсутствие. Что же касается того, как выбирать «верные направления», мы сейчас постараемся разобраться.

Частота тренировок

Как часто тренировать конкретную мышечную группу и как часто ходить на тренировки вообще? Слушайте основные моменты, друзья! У нас есть крупные мышцы (Спина, Ноги, Грудь) и мелкие мышцы (Дельты, Бицепс, Трицепс, Икры) . Первые восстанавливаются дольше, чем вторые из за своего размера. Иначе говоря, для крупных групп мышц может понадобится на один день больше отдыха, чем для мелких. Следующий важный момент – это объем приложенной работы во время тренировки. Чем тяжелее и дольше вы тренировали мышцу, тем более длительный отдых ей нужен. И последний важный момент – это ваша тренированность. Чем дольше вы тренируетесь, тем больше с одной стороны вы адаптированы переносить нагрузку и быстрее восстанавливаться. С другой стороны, чем больше у вас становятся ваши мышцы, тем больше времени им нужно для восстановления. Вот такой парадоксик. Причем, рост восстановительных возможностей будет отставать. К примеру, вы увеличили вашу руку с 30 см до 50 см…. Восстановительные возможности увеличились на 50%, а количество необходимого отдыха может увеличится на 200%. Эту информацию обычно не дают новичкам. Но я хочу, чтоб вы развивались быстро и уже сейчас задумывались бы над будущими достижениями, поэтому говорю о тех «подводных камнях», с которыми вы столкнетесь в своих будущих плаваниях.

Вообще, у каждого тренера существует свои представления о количестве отдыха между тренировками. Я, лично, считаю, что для новичков достаточно 2-4 суток между тренировками одной мышечной группы. Это не очень длительный отдых и я часто рекомендую его новичкам по двум причинам: во-первых, их мышцы достаточно «маленькие» и восстановятся быстрее, чем у профессионального бодибилдера, а во-вторых, новички сокращают свои мышцы с низким КПД (связь мозг-мышцы у новичка еще работает не настолько эффективно, как у опытного атлета).

Поэтому начинающим можно тренироваться достаточно часто с низкой или умеренной интенсивностью для того, чтоб привыкнуть к подобным нагрузкам и научить мышцы сокращаться с более высоким КПД.

Продолжительность Тренировки и Отдых между Подходами

Продолжительность тренировки чаще всего должна укладываться в 1 час, если вы тренируетесь натурально (т.е. без использования ААС). Почему? Просто потому что тренировка – это сильный стресс для вашего тела и гормональной системы. Тренировка – это всегда выброс кортизола и других катаболиков. Если затянуть время их высвобождения, то восстановление после этого стресса замедлиться в связи с замедлением выброса анаболических гормонов. В частности тестостерона. Естественно, если вы колите искусственные гормоны, то можете тренироваться дольше, т.к. на вас это не распространяется.

Отдых между подходами желательно делать в пределах одной минуты. Потому что подобный режим позволит вам сделать гораздо больший объем работы на тренировке, чем если бы вы отдыхали по 3 минуты между подходами. Именно в этом суть бодибилдинга, если сравнивать его с пауэрлифтингом. Мышцы больше всего растут от объемной работы с средними весами. А сила растет от низкообъемной работы с тяжелыми весами.

В общем, составляя тренировочную программу, на каждый подход прикидывайте 90-120 секунд (30 сек – подход и 60-90сек – отдых). В итоге, если тренировка у нас длиться 60 минут, вы можете в нее впихнуть максимум 30 подходов. Что достаточно много и является верхней границей. Обычно достаточно гораздо меньшего количества подходов.

Сплит и порядок тренировок различных мышц

Можно тренировать все тело за раз (как делали наши дедушки или как, бывает, рекомендуют новичкам), а можно расщеплять мышечные группы для тренировки в различные дни. Последний способ называется сплит (расщеплять) и считается более прогрессивным, потому что позволяет более сконцентрировано работать над каждой мышцей. С другой стороны, чем больше вы расщепляете ваше тело на различные дни, тем меньше у вас остается полных дней отдыха для восстановления. А рост мышц идет именно в дни отдыха. Это не так уж важно для больших парней, т.к. их мышцы могут восстанавливаться очень долго – 7-10 дней, но это может вредить начинающим, которым нужно тренировать группы чаще. Поэтому вот вам первое правило тренировочного сплита: Чем более вы зеленый, тем меньше расщепляйте тело по разным дням. Чем больше ваши мышцы, тем больше тренировочных дней вы можете делать в вашем комплексе.

Рад приветствовать, уважаемые читатели!

Все мы прекрасно знаем, что женщина любит ушами, а мужчина – глазами. Так вот, построение мускулистой и одновременно привлекательной фигуры – задача не из простых для любого пола, и тут (в приоритете) необходимо уделять внимание проработке наиболее привлекательных частей тела. Т.е. стоит сместить акцент в своих тренировках именно на самые “аппетитные” части, которые больше всего бросаются в глаза и позволяют выглядеть наиболее выигрышно на общем фоне.

Думаю, Вы уже догадались, что сегодня мы поговорим на тему — какие мышечные группы заставляют противоположный пол просто покрываться испариной и исходить истомой. Ну а главное, мы узнаем, как их сделать более “батонистыми”и рельефистыми.

Ну что, поехали…

Выигрышные мышечные группы. Какие они?

На первый взгляд может показаться странным, но женщины куда меньше реагируют на внешность мужчины и зачастую куда больше ценят чувство юмора, остроумие и уверенность мужчины в себе. Что касается женских предпочтений относительно мужской фигуры, то тут вкусы (от представительниц к представительнице) сильно разнятся. Одним нравятся поджарые модели с глянцевых журналов; другим – крупные и статные “мачомены”; третьи балдеют от своего соседа, офисного работника с узкими плечиками. Что касается конкретных частей тела, то здесь вкусы также разнятся: одни любят стальной мужской пресс, другим подавай округлые ягодицы, третьи – обеими руками за развитые плечи.

В общем у дам, как всегда – все сложно, то ли дело мужчины – их привлекает практически любая часть тела женщины. Нет, я не скажу, что остроумие и эффектность самой девушки не играют роли, просто они как-то сами собой отходят на второй план после нескольких минут общения. С чем это связано?…наверное, с матушкой природой.

Ну, мы немного отвлеклись от темы. Итак, какие мышечные группы должны быть в приоритете, чтобы выглядеть более привлекательно?

Тут стоит сказать, что мы должны следовать своим вкусам и предпочтениям в лепке своего тела, ведь оно ваше, и Вы его скульптор. Однако не стоит забывать, что человек — существо социальное, и то, что подумают о его фигуре окружающие, также имеет для него значение.

Примечание:

Вряд ли Арнольд Шварценеггер так усиленно тренировался, если бы жил не необитаемом острове, где некому было оценить его достижения.

Мужская уверенность в себе очень часто проистекает из хорошей физической формы собственного тела, именно она позволяет нравится себе и, как следствие, всем остальным (в частности — противоположному полу) . В одной из книг по биологии, помню, было сказано: “…самки предпочитают самцов, в чем-то выходящих за рамки обычных”.

Если применить сию мысль непосредственно к бодибилдингу, то построение мышц можно назвать чем-то выходящим за рамки обычного. Поэтому у любых мужских представителей, занимающихся в тренажерных залах, шансы привлечь внимание женщин существенно возрастают. Так все же, на каких следует предельно сфокусироваться, чтобы построить привлекательное и красивое тело?

Для мужчины – это грудная мышца, пресс, руки, ягодицы. Если Вы женщина, Вам необходимо просто держать себя в обычном мышечном тонусе, т.е. не “запускать” и следить за процентным соотношением жировой прослойки. Не помешают конечно фитнес-упражнения на ягодицы (для упругости) , грудь (для укрепления и поднятия) и пресс (плоский животик) .

Заметили, как много схожего? В целом, мышечные группы, которые делают наше тело визуально более притягательным, это: ягодицы, пресс, грудь и руки. Идем дальше и переходим к практической части.

Строим привлекательное и красивое тело. Лучшие упражнения.

Итак, начнем по порядку, пройдемся по каждой мышце и разберем, как же их лучше всего тренировать. И первая на очереди группа, это…

Ягодицы

Ягодицы, она же “филейная” часть. В связи с тем, что большую часть своего времени человек проводит на сидячей работе, то наиболее проблемной зоной многих женщин (также мужчин) как раз и является эта самая “труженица”.

Исходя из анатомических функций мышц ягодицы, становится понятным, что лучшими упражнения для ее тренировки являются различные приседания и выпады с гантелями. Более качественной проработки упругости ягодиц можно добиться, если объединить два упражнения (например, выпады и гиперэкстензию) в одну серию.

Чем хороши выпады?

Все дело в том, что это уникальное упражнение для проработки ног по отдельности, т.к. оно обеспечивает хорошую растяжку и концентрическое сокращение ягодичной мышцы. Все это происходит благодаря мощным нейромышечным импульсам, которые возникают при тренировке одной половины тела. Кроме того, в работу поочередно включаются такие крупные мышечные группы, как квадрицепсы и бицепсы бедер.

Почему именно гиперэкстензия?

Принято считать, что данное упражнение предназначено исключительно для проработки низа спины (точнее, разгибателей) , однако в первую очередь – это самый лучший изолятор ягодиц. Уж не знаю, в курсе ли девушки по поводу этой фишки, но в нашем зале это их любимый тренажер. Получается, что когда ноги выпрямлены, основными “приводяще-движущими” мышцами являются именно ягодицы, и поэтому нагрузка перераспределяется между разгибателями и большой (у кого как:)) ягодичной мышцей.

Итак, в целом можно придерживаться следующей тренировочной схемы (суперсет) и последовательности упражнений:

  • глубокие приседания (со штангой – для мужчин, с блином от грифа – для женщин) или жимы ногами в Гаккеншмидта (2-3 подхода по 8-10 повторений) ;

Потом суперсет:

  • выпады (2-3 Х 8-10 ) ;
  • гиперэкстензия (2-3 Х 10-12 ) ;

Примечание:

Суперсет – два разных упражнения в совмещенном подходе, без паузы и отдыха между ними.

Выполняйте эти упражнения, и тогда округлые ягодицы Вам обеспечены. Идем далее.

Грудные мышцы

Данная мышечная группа наиболее важна для женщин, т.к. от ее развитости зависит высота и упругость груди, а на эти параметры идет постоянное посягательство мужчин природы (вернее, гравитации – притяжение к Земле) . Также гармонично развитые грудные мышцы это большой плюс и для мужчин.

В целом, эти мышцы можно разделить на 3 секции: нижнюю, среднюю и верхнюю. Считается, что развить первые две не составляет большого труда, а вот над верхом придется попотеть. Опять же, наиболее прогрессивным вариантом будет объединение двух упражнений на разные секции в суперсет, тогда тренировочная схема будет выглядеть так:

Супер сет №1:

  • жим штанги/гантелей на наклонной (под углом вверх) скамье (2-3 по 10-12 повт.) ;
  • разведение/сведение в кроссовере (2 по 10 ) ;

Супер сет №2:

  • разведение гантелей на наклонной скамье (1-2 Х 8-10 ) ;
  • жим гантелей на скамье с обратным наклоном (1 по 8-10 ) .

Примечание:

Техника выполнения всех приведенных упражнений будет также разобрана в следующих статьях, так что подписывайтесь и следите за обновлениями.

Следующая мышечная группа, это…

Пресс


Пресс относится к группе “кора” – комплекс мышц, отвечающих за стабилизацию корпуса. Одним из самых распространенных заблуждений относительно мышц живота является то, что рельефный пресс – это всего лишь результат постоянных, тяжелых тренировок данной мышечной группы. Однако это неверно.

Вы можете “долбить” пресс хоть 5-6 раз в неделю, но результат не будет виден, т.к. заветные кубики наглухо “запечатаны” под жировой прослойкой. Поэтому для визуального эффекта – четкой прорисовки рельефа — откажитесь от балластных продуктов, сократите количество калорий и займитесь кардионагрузкой.

Для эффективной тренировки мышц брюшного пресса необходимо следовать следующей стратегии:

  • выполнять не более 15-20 повторений в одном подходе;
  • постепенно увеличивать сопротивление (вес отягощения) ;
  • давать отдых и тренировать пресс не более 1-2 раз в неделю;
  • сначала выполнять упражнения на нижний отдел пресса (поясню относительно последнего пункта) .

Все дело в том, что когда мы выполняем упражнения на низ пресса, также частичную нагрузку получает и верхний отдел. Если же сначала делать обычные кранчи (горизонтальные скручивания корпуса лежа) , а затем подъемы ног в висе, то получается, что утомившийся верх сдается раньше низа, таким образом, последний не дополучает нагрузку. Поэтому сначала лучше прорабатывать нижний отдел.

Тренировочная программа точеного пресса может выглядеть следующим образом:

  • подъем прямых ног/коленей в висе на перекладине/шведской стенке (2-3 Х 10-15 повторений) ;
  • обычные кранчи на абдоминальной скамье (аналогично) .

Последняя из топ 4 самых привлекательных групп – это…

Руки


Какой мужчина не любит быстрой езды мечтает о больших (наполненных) руках, и какая женщина не хочет иметь подтянутые, слегка мускулистые ручки?

И хотя, казалось бы, что цели совершенно разные, однако тренировочные программы подходят совершенно одинаковые, и добиться они позволяют нужных каждому результатов. Т.е. женщинам не стоит бояться того, что они накачают себе огромные руки – нет. Здесь все дело в природе и физиологии организма, а точнее — в разном количестве тестостерона, плотности мышечных волокон у женщин и мужчин.

Таким образом, конечная программа может выглядеть следующим образом:

Суперсет (трицепсы) :

  • фрацузский жим лежа на горизонтальной скамье (2 подх. по 8-10 ) ;
  • узким хватом (аналогично) .

Суперсет (бицепс) :

  • сгибание рук с гантелями сидя (2 подхода по 10-12 повторений) ;
  • сгибание рук на скамье Скотта (то же самое) ;

Упражнения выполняются в “темпе вальса” без пауз и отдыха, т. е. желательно иметь снаряды под рукой, а не бегать к ним через весь зал. Собственно, программы тренировки самых привлекательных частей тела мы рассмотрели, осталось подвести некоторые итоги.

Итак, все представленные схемы работают по принципу суперсета – соединение двух упражнений в одно. Хочется сказать, что это отличный способ быстрого достижения результатов, т.к. проводится большая мышечная работа за меньший промежуток времени. Кроме того, данный принцип (суперсета) способствует повышенной секретизации гормона роста, что, в свою очередь, позитивно сказывается на процессах построения мышц и сжигания жира. Также этот гормон повышает общий тонус кожи, что тоже сказывается на внешности.

Послесловие

Сегодня мы узнали про самые “аппетитные” части тела, и то, как их нужно тренировать. Я желаю, чтобы Ваша фигура обзавелась как можно большим количеством привлекательных деталей уже в самом скором времени!

До встречи, заходите почаще, здесь Вам всегда рады!

PS. Уважаемые читатели, а какие для Вас (в противоположном поле) самые привлекательные части тела? Делитесь своими соображениями в комментариях.

На фото Арнольд Шварценеггер

Вконтакте

Одноклассники

Любой мужчина, который хочет, чтобы его уважали в обществе не только за его ум, часто задумывается о своих физических данных. Однако одно дело думать об этом, а другое дело начать.

Эта статья самым непосредственным образом расскажет о том, как именно нужно действовать, чтобы Ваш прогресс в фитнесе поразил окружающих.

Где заниматься — дома или в зале?

Часто у людей, далеких от спорта, появляется вопрос: где лучше тренироваться, в спортивном зале или дома? Ответ зависит от Ваших целей. Если задачей является приобретение объемной гармоничной мускулатуры, то путь один — фитнес-центр или спортивный зал.

А если Вы хотите просто немного изменить свое телосложение, то подойдут и домашние тренировки, но без дополнительного веса прогресс быстро остановится, а покупка домой гантели и штанги обойдется едва ли дешевле, чем абонемент в зал.

Поэтому после занятий дома для того, чтобы продолжать расти и развиваться в любом случае нужно идти в спортивный клуб. А есть ли тогда смысл терять время дома, когда можно будет заниматься в специально оборудованном центре с куда большим комфортом и прогрессом?

Как начать тренироваться?

Для начала тренировок следует точно поставить себе цель, ведь правильно подобранная мотивация — это уже половина победы.

Большинство читателей не нужно мотивировать дополнительно, потому что главная мотивация — отражение в зеркале. И если кто-то скажет Вам, что не хотел бы набрать несколько килограмм качественной мышечной массы, то, поверьте, он как минимум лукавит.

Занятия следует начать с обследования у доктора, если Вы этого не делали раньше. Ведь, возможно, некоторые упражнения следует исключить из тренировочной программы (о составлении программы расскажем ниже).

Такое иногда случается, например, при травмах спины или шеи, но даже если у Вас есть такие проблемы, то не расстраивайтесь, упражнений сейчас существует огромное количество и существуют разные способы работы на определенные мышцы, которые можно использовать даже при травмах.

Когда первый этап пройден, пора переходить к покупке абонемента. Тут не будет идеального совета, но все же лучше посещать спортивный зал, который находится рядом с домом, работой или учебой, ведь после тренировки очень важен отдых.

На данный момент сейчас такое обилие фитнес-центров, что любой читатель найдет себе зал по душе. Единственный практический совет, который можно дать: не скупитесь на своем здоровье. Ведь за редким исключением цена прямо пропорциональна качеству предоставляемых услуг.

Согласитесь, поплавать в теплом бассейне и посетить после тренировки не только очень приятно, но еще и полезно.

Разминка

Каким видом спорта Вы бы не занимались, следует запомнить одну очень важную вещь, без которой тренировки не только не принесут результата, но еще и могут навредить здоровью. Речь идет о разминке.

Ведь тяжелые веса, без которых немыслима тренировка для набора мышечной массы, разрушающе повлияют на неразогретые мышцы и суставы.

Согласитесь, обидно будет получить серьезную травму из-за того, что захотелось сэкономить 5-10 минут на простых разогревающих упражнениях.

Как именно разминаться знают все еще со школьных уроков физкультуры: простые движения, начинающиеся с верхней части тела.

Также перед каждым упражнением следует сделать несколько подходов с небольшими весами, постепенно повышая вес до рабочего. Это прекрасно разогреет мышцы и суставы и защитит от травм.

Программа тренировок как правильно качать мышцы

Вот Вы купили абонемент, приступили к тренировкам. Главной ошибкой начинающих на этом пункте является совершенно непродуманная программа тренировок. На данный момент существует множество программ как правильно качать мышцы тела и каждая из них имеет свои преимущества.

На начальном этапе больше всего подойдет такая схема: 1-ая тренировка: ноги, плечи, 2-ая: грудь, трицпсы, 3-я спина, бицепсы.

Бросается в глаза отсутствие конкретных упражнений, написаны только группы мышц, которые следует нагружать в каждую тренировку, и это неспроста.

Существует огромнейшее количество упражнений на каждую группу мышц, но на начальном этапе свое внимание следует обратить на несколько главных упражнений, благодаря которым с Вашим телом при правильном выполнении и питании будут происходить настоящие чудеса.

И так, начать следует со «святой» для фитнесистов и бодибилдеров тройки упражнений. А именно: жим лежа, присед со штангой на плечах и становая тяга. Если у Вас есть проблемы со спиной, то от двух последних упражнений придется отказаться, либо выполнять их очень аккуратно.

Возникает вопрос, почему же именно эти три упражнения так влияют на рост мускулатуры? Ведь те же мышцы можно нагрузить в тренажерах. Ответ уже давно дали ученые, занимающиеся биохимией.

В этих упражнениях включаются в работу почти все группы мышц, реакция организма не заставляет себя долго ждать. Начинает выделяться огромное количество тестостерона, гормона, который делает мужчину мужчиной, который в разы ускоряет синтез белка в мышцах.

Конечно, не стоит забывать и об остальных упражнениях, но большую часть прогресса дадут именно базовые упражнения.

Остальные упражнения, которые будут хороши как на начальном этапе, так и на протяжении всех тренировок:

    жим штанги или гантелей над головой — плечи

    подъем штанги или гантелей на бицепс

    подтягивания, тяга блоков — спина

    жим узким хватом — трицепс

От правильной техники выполнения упражнений зависит абсолютно все в ваших тренировках. А именно прогресс и отсутствие травм. Что следует запомнить раз и на всегда, так это то, что спину во всех упражнениях следует держать прямой.

Второе, что нужно помнить — никогда не позволяйте себе читерить, если только это не последний подход . Что под этим подразумевается? Например, раскачивание при подъеме штанги на бицепс.

В идеале это упражнение стоит делать стоя у стены, прижавшись лопатками и тазом. В этот список можно включить и разведение локтей в стороны при жиме узким хватом.

Читинг — неправильные движения во время упражнения, которые заставляют работать совершенно не те мышцы, которые нужно, но позволяют поднимать большие веса.

Использовать его целесообразно в конце тренировки, чтобы окончательно «добить» определенные группы мышц.

Сколько тренироваться в неделю


Часто новичок жалуется на отсутствие прогресса. А в ответ на вопрос «Как часто ты тренируешься» спокойно говорит «Каждый день». И все становится понятно.

Всем давно известно, что мышцы растут не во время тренировок, а во время отдыха, особенно сна. Поэтому если не давать отдыха своему организму, то вместо роста мышц (анаболизма) начинается совершенно иной процесс — катаболизм.

Так называется разрушение белка в мышцах, это приводит только к потере объемов, которые Вы набирали так старательно.

После тренировки организму нужно давать отдых. Как минимум это будет 1 день, но оптимальный вариант — 2 дня отдыха между каждой тренировкой. Так организм будет успевать восстанавливаться в полной мере, и занимающийся будет подходить к каждой тренировке в идеальной форме.

Мечтаете о развитой мускулатуре? В данной статье рассматриваются упражнения для набора мышечной массы.

Привет всем, уважаемые читатели моего блога. Сегодня я рассказываю о том, какие мышцы можно и нужно тренировать вместе, что бы активировать выброс анаболических гормонов и ускорить рост наших мышц.

На сегодняшний день в бодибилдинге есть прочное убеждение о том, что на одной тренировке можно тренировать не менее 2-3 мышечных групп. Профессионалы, кстати, бомбятся вообще по одной группе в день. Но мы не профи, мы будем работать в другом ритме.

В следующей статье я расскажу вам, как возможно относительно быстро похудеть и сбросить вес. Тема будет особенно интересна девушкам, в преддверии пляжного сезона. Да и мужикам будет полезно подсушиться, у кого проблемы с этим. Не забудьте подписаться на выход новых статей моего блога, что бы не пропустить этот пост.

Когда проводили исследования, то выяснили, что максимальный эффект достигается в двух случаях.

  1. Когда вы тренируете только большие мышечные группы. Спину, грудь и ноги.
  2. Когда вы совмещаете тренировку большой мышечной группы с тренировкой малой, сопутствующей мышечной группы. Например, совмещаете тренировку груди с тренировкой трицепса, тренировка плеч, сочетается с тренингом ног, а тренировка спины очень хорошо сочетается с тренировкой бицепса.

В первом случае вы просто интенсивно тренируете большие мышечные группы, которые как локомотив тянут за собой всё тело. Упражнения тут вы выбираете тяжёлые, базовые. Жимы, тяги. Приседы и так далее. Тяжёлая база по полной программе нагружает и плечи и мышцы рук, так как все жимы и тяги выполняются и с их помощью то же. Поэтому развивая грудь, вы обязательно будет иметь и сильные руки то же.

Во втором случае мы бомбим сопутствующую мышечную группу дополнительными упражнениями и вызываем дополнительный её рост. Главное правило тут тренировать вначале большую мышечную группу, а потом мышцу сопутствующую. Например, вначале работаем грудь, потом трицепс. Мышцы рук в этом случае нам нужны свежие, что бы прокачать большую и более важную для нас мышечную группу груди. А трицепс мы уже дорабатываем после.

Такая же схема применима к тренировкам спины-бицепса, и ног-плеч. Вначале работаем большую группу, потом сопутствующую.

Количество подходов и повторов выбирайте сами в зависимости от вашего телосложения и обмена веществ. Если вы мезоморф, то 3-4 подхода с 10-12 повторами для вас будет самое оно. А если вы, к примеру, эктоморф, тогда ваша схема – 2 подхода с 8-10 повторами.

Можно привести в качестве примера такую схему тренировки.

Понедельник

Грудь

Трицепс

Пресс

Среда

Ноги

Плечи

Пресс

Пятница

Спина

Бицепс

Пресс

Как видите, пресс мы тренируем каждую тренировку. На него достаточно будет 2 упражнений за тренировку. Прямые+нижние, или прямые+ косые, к примеру.

Если же вы имеете возможность тренироваться только два раза в неделю, тогда логичнее будет тренировать только большие мышцы плюс какую-то одну малую. Схема может выглядеть так.

Понедельник

Грудь

Спина

Пресс

Четверг

Ноги

Плечи

Пресс

Упражнения советую выбирать базовые, в особенности, если у вас цель накачать массу. Количество повторов и сетов – выбирайте сами исходя из вашего телосложения, вашей подготовленности и возможности вашего кошелька покупать качественный «спортфуд».

Опять же повторяюсь. Не забывайте о питании и грамотном отдыхе. Любая тренировка должна компенсироваться и поддерживаться питанием и отдыхом. Чем жёстче тренировка, тем обильнее и качественнее должен быть отдых и питание растренированных мышц. Вносите изменения в свой тренировочный режим – это, пожалуй, самое важное для предотвращения застоя в результатах и прогрессе.

В качестве учебного пособия для самостоятельных тренировок могу порекомендовать Бесплатный курс о наборе массы . Посмотрев 5 видео уроков вы узнаете, как избежать основных ошибок, при наборе массы, как накачать красивую грудь и составить свою программу питания для набора мышечной массы в процессе тренировок.

Для тех, кто хочет поучить более продвинутые методики тренинга, рекомендую ознакомится с платным тренингом «Базовый курс на массу» Автор Владимир Молодов, бодибилдер с 11-ти летним стажем. В курсе ещё больше фишек и приёмов, которые помогут вам набрать массу всего за 90 дней. И отзывы это подтверждают.

А на сегодня у меня всё, до встречи в следующей статье про похудение! Берегите себя.

Анекдот дня.

Что это у тебя с лицом?

Вчера ходил в тренажерный зал, хотел себя в форму привести…

При чем тут тренажерный зал, я тебя про лицо спрашиваю. Что с лицом???

Так в этом зале вчера один качок гирю уронил…

Нет! Себе на ногу !

А лицо?

А мое лицо решило над этим посмеяться…

P/S Не нужно сдерживать эмоции, идущие от сердца! Выплесни их в комментариях! Будет интересно узнать, что ты думаешь по поводу прочитанного!
Не забудьте подписаться на обновление блога , что бы первым получать свежие статьи!

✅ Какие мышцы нужно качать чтобы. Какие группы мышц лучше тренировать вместе

Как совмещать мышечные группы на тренировке?

Содержание статьи:

  1. Какие нужно тренировать вместе
  2. Какие нужно тренировать в первую очередь

Многие начинающие атлеты не удовлетворены получаемыми на тренировках результатами. К слову, именно невысокая скорость прогресса или его полное отсутствие толкает человека на использование спортивной фармакологии. Если в профессиональном спорте без этого не обойтись, то на любительском уровне очень даже можно, ведь в приоритете должно быть здоровье, а не размеры мускулов.

Однако мы отвлеклись, и сегодняшняя статья ответит на популярный вопрос, какие мышцы нужно тренировать вместе и в первую очередь? Вариантов проведения занятий много и все зависит от ваших приоритетов, поставленных задач, тренировочного стажа и т. д. Сейчас мы рассмотрим наиболее популярные варианты совмещения тренинга мускульных групп. Начнем с самого простого и завершим сложной системой, которая подойдет тем, кто планирует добиться высоких результатов и возможно принимать участие в состязаниях.

Какие мышцы нужно тренировать вместе?

По мере роста вашего уровня подготовки, станет понятно, что необходимо увеличивать интенсивность занятий и вводит в тренировочную программу дополнительные упражнения. Сейчас речь пойдет о возможных и наиболее популярных совмещениях в тренинге мускульных групп. Рекомендуем вам начинать с самого простого и постепенно двигаться вперед.

Фулбоди

Данный вид совмещения предполагает тренировку всех мускулов тела на каждом занятии. Эту систему стоит использовать начинающим атлетам, выполняя два или три сета в каждом движении. Не стоит пугаться сравнительно низкой интенсивности таких занятий. Если вы прежде не занимались спортом, то организму требуется время на адаптацию к новым условиям жизни. Если вы соблюдаете все принципы культуризма, то на первом этапе фулбоди станет отличным выбором, и вы будете достаточно быстро прогрессировать.

Первое время вашей основной задачей станет обучение организма рациональному использованию собственных возможностей. Также напомним, что первый месяц тренинга вам вовсе стоит посвятить освоению технических нюансов выполнения основных упражнений. Не стоит сразу прогрессировать рабочий вес, ведь если нарушается техника выполнения движений, то они не будут эффективными.

Занятия следует проводить трижды в неделю, предоставляя организму около 48 часов на восстановление. Еще одной причиной невысокой интенсивности тренинга на начальном этапе занятия является возможность минимизировать болевые ощущения в мускулах после завершения тренировки. Выбирайте на каждую мускульную группу по одному базовому упражнению и выполняйте его в двух или трех сетах. Число повторов в каждом подходе должно составлять от 10 до 12.

Верх-низ

Данная система также называется двухдневный сплит. Рекомендуем переходить на нее после фулбоди. Суть системы проста — тело разбивается на верх и низ. После этого на одном занятии вы прорабатываете мускулы верхней части тела, а на втором качаете ноги. На каждую группу предстоит выполнять уже по два движения. Причем в течение недели можно тренироваться уже пять дней. В каждом упражнении выполняйте три сета, а число повторов может быть следующим:

  • Для набора массы — от 10 до 12.
  • Для повышения силовых параметров — от 6 до 8.

Трёхдневный сплит

Данная система тренинга станет для вас следующим шагом на пути к созданию привлекательного тела. Каждая мускульная группа на протяжении недели будет прокачиваться уже трижды. Наиболее простым разделением тела на группы будет следующий принцип:

  • Тянущие мускулы.
  • Толкающие мускулы.
  • Ноги.

В результате вам предстоит каждое занятие чередовать тренинг следующих мышц:

  • Плечи, грудь и трицепс.
  • Ноги.
  • Бицепс и спина.

Число повторов аналогично предыдущей схеме, а количество сетов в каждом упражнении для ног стоит увеличить до четырёх.

Четырехднёвный сплит

Это уже достаточно серьезная система тренинга, предполагающая значительное увеличение интенсивности занятий. Выполняя четырехдневный сплит на протяжении недели, каждая мускульная группа получит для восстановления около 72 часов. Расписание тренинга может выглядеть следующим образом:

  • 1-й день — бицепс и спина.
  • 2-й день — трицепс и грудь.
  • 3-й день — отдых.
  • 4-й день — ноги.
  • 5-й день — плечи.
  • 6-й и 7-й дни — отдых.

На каждую группу рекомендуем выбирать три или четыре движения, а для прокачки ног стоит выполнять пять. Количество сетов равно трем или четырем, а повторов — 6–15.

Пятидневный сплит

Данная система станет отличным выбором для опытных атлетов, которых можно назвать фанатами тренажерного зала. Она весьма похожа на предыдущий сплит, просто тело разделяется не на четыре, а пять групп. Вы можете на протяжении недели чередовать тренинг в следующем порядке: спина, грудь, ноги, плечи, трицепс совместно с бицепсом. Два оставшихся дня предназначены для отдыха. Возможно, вы обратили внимание, что ни в одной из систем мы не упоминали о прессе и икроножных мускулов. Это связано с тем, что мышцы малого размера восстанавливаются быстро, и вы можете работать над ними через день.

Какие мышцы нужно тренировать в первую очередь?

Сегодняшнюю тему, какие мышцы нужно тренировать вместе и в первую очередь, мы решили разделить на две части. Вы уже знаете принципы построения сплитов. По большому счету, тело необходимо прокачивать гармонично. Однако в зависимости от гендерной принадлежности люди стараются отдать предпочтение то или иной группе мышц. Вполне очевидно, что девушки особое внимание уделяют ногам и особенно ягодицам. Для парней более важными являются грудь, руки и пресс. О правилах тренировки этих мускулов сейчас и пойдет речь.

Ягодицы

Сегодня многие рабочие профессии связаны с сидячей работой. Это крайне негативно сказывается на состоянии мускулов ягодиц. Причем данное утверждение справедливо не только для женщин, но и мужчин. Другое дело, что девушки во время тренинга значительно больше внимания уделяют работе над этой частью тела. Парням в принципе достаточно просто тренировать ноги. А вот девушкам, желающим иметь упругие ягодицы аппетитной формы, нижеследующая информация будет весьма полезна.

Отличным решением для вас станет объединений гиперэкстензии и выпадов в одну серию. Выпады можно считать одним из лучших движения для мускулов ягодиц. Это связано не только с их активной работой, но и качественной растяжкой. Одновременно прорабатываются квадрицепс и бицепс бедра. Гиперэкстензия в свою очередь позволяет обеспечить изолированную нагрузку именно на ягодицы.

Девушкам тренинг ягодиц стоит начать с выполнения глубоких приседаний либо жимов ногами в тренажере Гаккеншмидта. Сделайте два или три сета по 8–10 повторов. После этого наступает очередь суперсета:

  1. Выпады — два или три сета при 8–10 повторах в каждом.
  2. Гиперэкстензия — 2–3 сета при 8–10 повторах в каждом.

Напомним, что суперсет предполагает выполнение двух движений без паузы между ними.

Грудь

Не стоит считать, что грудные мускулы имеют важное значение только для мужчин. Девушки также должны уделять данной группе внимание, ведь она позволяет приподнять бюст и сделать ее более упругой. Мускулы груди можно условно разделить на три отдела: средний, верхний и нижний. Принято считать, что наиболее сложно качественно прокачать верхний отдел.

Мы рекомендуем для получения хороших результатов использовать два суперсета.

В состав первого войдут следующие движения:

    Жимы гантелей (штанги) на наклонной скамейке (угол направлен вверх) — два или три сета при 1012 повторах в каждом.
  • Разведение-сведение рук в кроссовере — два сета по 10 повторов в каждом.
  • Второй суперсет выглядит следующим образом:

    1. Разводка гантелей на наклонной скамейке — один или два сета по 8–10 повторов.
    2. Жимы гантелей на наклонной скамейке, обратный наклон — один сет при 8–10 повторах в каждом.

    Мускулы живота

    Пресс входит в мускульную группу кора, которая отвечает за стабилизацию корпуса. Многие атлеты уверен, что чем активнее они будут прокачивать пресс, тем быстрее появятся долгожданные кубики. Но вы должны помнить, что пресс будет выглядеть красиво только в том случае, если в области живота будет находиться минимальное количество жировых отложений. Это говорит о том, что креме тренировок вам пристальное внимание необходимо уделять и программе питания.

    Чтобы тренинг мускулов пресса дал результат, рекомендуем воспользоваться несколькими простыми советами:

      В каждом сете необходимо выполнять от 15 до 20 повторов и не более.

    Необходимо прогрессировать нагрузку, используя для этого отягощения.

    Работайте над мускульной группой один или два раза в неделю, ведь мышцы должны успеть восстановиться.

  • Сначала работайте над нижним отделом пресса.
  • Последний пункт требует разъяснений. Как вам вероятно известно, пресс представляет собой один мускул и в принципе, его не стоит разделять на отделы. Говоря проще, при выполнении любого движения прорабатывается весь пресс. Однако если вы сначала будете выполнять, скажем, классические скручивания, а затем подъемы ног в висе, то верх пресса устанет раньше и низ не получит достаточной нагрузки, рекомендуем поступить следующим образом:

    1. Подъёмы ног в висе — два или три сета по 10–15 повторов в каждом.
    2. Классические скручивания с аналогичной интенсивностью.

    Еще одна мускульная группа, которая имеет важное значение не только для мужчин, но и женщин. Если с парнями все понятно, то прокачивая руки, девушки смогут устранить обвисания кожного покрова. Не смотря на некоторые отличия в физиологии, существующие между полами, принципы тренинга остаются неизменными.

    В этой связи хочется еще раз напомнить милым дамам, что не стоит бояться перекачать ручки. Это просто невозможно по причине минимального содержания тестостерона в женском организме. Кроме этого есть отличия и в композиции мышечных волокон.

    Для получения хорошего результата снова рекомендуем два суперсета. Первый из них имеет следующий вид:

    1. Французские жимы на горизонтальной скамейке — два сета по 8–10 повторов в каждом.
    2. Жимы штанги в положении лежа, узкий хват — аналогичная интенсивность.

    Второй суперсет включает следующие движения:

    1. Сгибание рук с гантелями в положении сидя — два сета по 10–12 повторов.
    2. Сгибание рук на скамейке Скотта — интенсивность аналогична первому движению.

    Перед началом выполнения суперсетов вам необходимо подготовить всё необходимые спортивные снаряды, чтобы не бегать за ними. В противном случае весь смысл суперсета исчезнет. Заметим, что суперсеты являются эффективным способом увеличения интенсивности тренинга. Вот собственно и вся информация, с которой мы хотели поделиться при ответе на вопрос, какие мышцы нужно тренировать вместе и в первую очередь?

    Больше о том, какие мышцы нужно тренировать вместе и какие в первую очередь:

    Какие группы мышц лучше тренировать вместе

    Новичок в силовом спорте? Тогда построение своего графика тренировок имеет первостепенную важность. Далее Вы узнаете, какие группы мышц лучше совмещать в тренировочный день, чтобы максимизировать мышечный рост.

    В бодибилдинге никто и никогда не предоставит Вам программу тренировок, которая волшебным образом превратит Вас в успешного соревнующегося атлета. Эта занимает не один год упорного труда, проб и ошибок, чтобы заполучить тело своей мечты. Здесь Вы можете найти лишь рекомендации о лучших упражнениях, успешных схемах по количеству подходов и повторений, интересных способах тренинга, но в итоге только Вы являетесь и «жюри» и «судьей» в вопросе эффективности тех или иных методах тренировок для своего тела.

    Итак, выстраивание своей тренировки и тренировочного сплита завязано на нескольких факторах:

    Новичкам следует придерживаться менее объемным и интенсивным программам, но с большей частотой, чем опытным спортсменам.

    Собираетесь ли Вы просто держать мышцы в тонусе или, может быть, хотите произвести более масштабные изменения своего телосложения и достигнуть новых вершин?

    Сможете ли Вы тренироваться 5 дней в неделю, или же Ваш график настолько плотный, что Вы можете позволить себе посещать тренажерный зал лишь пару дней? Как бы то ни было, важно осознать, что каждая последующая тренировка основывается на предыдущей. Так что у Вас должна быть возможность посещать зал хотя бы 3 раза в неделю.

    Отдых и восстановление

    В зависимости от Вашей работы, образа жизни, а также восстановительных способностей (включая сон), может потребоваться больше или меньше дней отдыха между тренингом. И не стоит им пренебрегать в погоне за мышечной массой. Рост происходит за пределами тренажерного зала, при наличии хорошего питания и полноценного восстановления. Обязательно прислушивайтесь к своему организму.

    Восстановление может также включать в себя умственную перезарядку — если морально устали от регулярных тренировок, возьмите чуть больше дней для отдыха. Одну из тренировок можно провести вне тренажерного зала, сделав забег на длинную дистанцию или поработав на турнике и брусьях.

    Если у Вас есть отстающие группы мышц или те, которые Вы просто хотели бы подтянуть, работайте над ними в первую очередь после периода отдыха, когда запасы энергии находятся в избытке.

    Какие совмещения существуют

    Есть множество вариантов тренировок, разделяя прокачку или же совмещая группы мышц в один день. Ниже представлены 5 основных тренировочных сплита, начиная с самого легкого и заканчивая самым продвинутым и сложным. Не ошибитесь! Новичкам следует выбрать первый вариант, тогда как более менее матерые атлеты могут рассмотреть остальные вариации.

    Становясь опытнее, приобретая новые знания и навыки, Вы обнаружите, что начинаете использовать больше упражнений, а интенсивность и объем Ваших тренировок растет. Все это потребует больше времени для отдыха, а значит каждая группа мышц будет прорабатываться один раз в неделю. Итак, какие мышцы лучше тренировать вместе?

    Совмещение всех групп мышц за одну тренировку — наилучший вариант для новичков, основывающаяся на выполнении 2-3 подходов одного упражнения на мышечную группу. Главная причина, почему объем тренировки (мало подходов на группу) невысок — первичная адаптация новичков к силовым нагрузкам проходит через нервную систему. Ведь сначала нужно научить свой организм активировать и использовать как можно больше мышечных волокон, а уже потом работать над их силой и размерами. Это в свою очередь требует большей частоты, а тренировка должна повторяться 3 раза в неделю с 48 часовым восстановлением в промежутке.

    Другая причина, по которой объем и интенсивность тренировок удерживаются на низком уровне для новичка — это сведение к минимуму на следующий день боли в мышцах после тренировки. Прокачка всего тела помогает не только познакомится со всем оборудованием и тренажерами, но и позволяет прорабатывать каждую группу мышц в умеренных объемах, вместо «выжигания» одной конкретной.

    Какие группы мышц лучше совмещать в тренировочный день?

    Тренировочная программа зависит от уровня занимающегося, его целей, и особенностей строения мышц.

    Строим программу на основе опыта

    Новички должны делать акцент на улучшении общих физических показателей. Сила, выносливость, подвижность суставов и гибкость важнее для их роста, чем упор на «отстающие дельты». Старая школа бодибилдинга исходит из того, что физически слабый атлет с недостаточно развитыми двигательными навыками не сможет построить мышцы при натуральном тренинге.

    Этапы подготовки новичка можно представить как:

    • Отработка техники;
    • Набор мышечной массы;
    • Сжигание лишнего жира.

    В модном фитнес-центре тренер будет убеждать клиента, что делает с ним именно то, ради чего он пришел. Девушке скажет, что та похудеет от приседаний на ящик и тяг с легким весом, худому парню – что он наберет от отжиманий с паузой и подтягиваний. Но истина в том, что пока первичный двигательный навык не будет получен, придется развивать все тело на каждой тренировке.

    Фулбоди подходит новичкам потому, что позволяет при 3 тренировках в неделю два раза прорабатывать приседание, 1-2 раза тягу, подтягиваться и отжиматься через день, укреплять мышцы кора и разгибатели спины на каждой тренировке. Начинающий бодибилдер не будет травмироваться из-за высоких весов отягощения, но отработает технику выполнения базовых упражнений.

    Мышцы разгибатели спины

    Кроме того, фулбоди поможет ему быстрее сбросить лишний вес, так как позволяет расходовать больше энергии в единицу времени. Не все люди сразу могут приступить к базовым упражнениям, некоторым нужен вводный курс физкультуры: бег в невысоком темпе, статические упражнения со своим весом, например планка или облегчённые варианты бёрпи.

    Когда можно считать себя «опытным»? Ответ на этот вопрос индивидуален. Для целей построения сплита, стоит переходить на тренировки отдельных групп мышц в разные дни, когда техника базовых упражнений сформируется, человек начнет двигаться, не задумываясь. Есть и субъективный критерий – проходит 3 месяца от начала занятий.

    В практике тренерской деятельности ориентиром служит именно 3-4 месячный цикл. Если клиент посещает занятия без пропусков, к этому моменту его мышцы, связки и ЦНС готовы перенести классический сплит.

    Частота тренировок и их планирование

    Многие могут посещать зал только 2 раза в неделю. В этом случае сначала рекомендуют делать 2 тренировки на все тело, причем с разным набором упражнений:

    Первая – коленно – доминантные (присед, жим ногами, выпады) упражнения для ног, жимы для верха тела, и тренировка кора.

    Вторая – тазово – доминантные упражнения для ног (тяги и наклоны со штангой), тяги для спины (в наклоне, если возможно – то и становая в классической постановке), и упражнения на бицепс.

    Фулбоди VS сплит

    По прошествии 2-3 месяцев, и достижении предела в весах отягощений, можно перейти на сплит по принципу «верх-низ». Чего не нужно делать, так это отказываться от работы с мышцами ног. Некоторые перестают качать ноги, чтобы больше времени оставалось на грудь и спину, но это неправильно. Ноги – мощный упор в жимах, и важнейший рычаг для подъема веса. Их недостаточное развитие может стать причиной травмы как в быту, так и в зале. Есть и теория, что анаболический фон в организме сложно создать, если не делать базовые упражнения.

    При трехдневном посещении зала обычно чередуют тренировки 1 и 2 в варианте фулбоди, без привязки к неделе. По прошествии времени переходят на один из классических сплитов:

    • Грудь-трицепс, спина-бицепс, ноги-дельты, пресс на каждой тренировке;
    • Грудь-бицепс, спина-трицепс, ноги-дельты, пресс.

    Слабые и сильные стороны спортсмена

    В бодибилдинге принято считать слабыми сторонами те группы мышц, которые плохо растут. А в силовых видах спорта – недостатки двигательного паттерна (привычки) и особенности строения тела, которые не позволяют поставить адекватную технику.

    Пример: у спортсменки длинные ноги, бедра, узкий таз, и длинные руки. Ее цель – накачать ягодицы, сохраняя низкий процент жира. С точки зрения силовой работы, ей будет сложнее поставить технику приседания так, чтобы была достигнута полная амплитуда работы. С точки зрения бодибилдинга у нее могут как отставать, так и нормально откликаться ягодицы, ведь есть много других упражнений, кроме приседа.

    При составлении сплита новичку исходят из того, что у него есть слабые двигательные навыки, а не «отстающие мышцы». Потому в некоторых случаях может быть и 3 приседания в неделю, при 1 тяге, и прокачке спины каждый день.

    Продвинутые атлеты должны прорабатывать сначала отстающие группы мышц. Делать «специализацию», то есть ставить в тренировочный план прокачку каких-то групп 2 раза в неделю имеет смысл при адекватном режиме дня, хорошем восстановлении, и развитых двигательных навыках.

    Отдых и восстановление

    В идеале, тренировка должна приходиться на пик суперкомпенсации. Он наступает через 36-72 часа после основной силовой работы. Новичку целесообразно тренировать все тело через день потому, что он не использует в своих занятиях высокие веса отягощений, способные значительно повлиять на состояние опорно-двигательного аппарата. В таком тренинге каждая крупная группа мышц прокачивается двумя, максимум, тремя упражнениями. Объем не превышает 6-12 рабочих подходов на группу.

    Более продвинутые клиенты могут тренировать группу мышц раз в неделю, так как выполняют больший объем работы. Они обычно выполняют до 5 упражнений, некоторые – больше.

    Виды совмещения

    Упражнения в тренировочной программе можно совмещать:

    • Фулбоди – проработка всего тела;
    • Сплит – группа мышц в тренировочный день;
    • Отработка навыков – тренировка строится вокруг упражнения, и нужных для адекватных силовых в нём – вспомогательных движений.

    Важно: последний подход оправдан с точки зрения биомеханики больше, чем бодибилдерские сплиты, западные обучающие организации (NASM, ISSA) исходят из того, что с фитнес-клиентом тренер будет заниматься отработкой навыков.

    В сплитах тоже нет ничего плохого, они прекрасно помогают набрать мышечную массу и построить атлетичное тело. Для раскрытия потенциала необходимо учитывать индивидуальные особенности восстановления.

    Фулбоди

    Вариантов проработки всего тела несколько:

    • Изучение базовых упражнений. Берется приседание или тяга, и к нему добавляются жим лежа, либо стоя, простое или австралийское подтягивание, и какие-нибудь легкие махи на плечи, и сгибания на бицепс. Упражнения чередуются по дням, адекватная разбивка была рассмотрена выше;
    • Укрепление мышц и связок человека с «двигательными проблемам», На каждой тренировке человек выполняет одинаковый набор упражнений в тренажерах. Обычно это жим ногами, гиперэкстензия, жимы в тренажерах на грудь и плечи, тяга блочного тренажера на спину или подтягивание в гравитроне, тяга к поясу сидя, и упражнения на пресс;
    • Фулбоди с «акцентом» на будущие занятия кроссфитом или функциональным многоборьем. К изучению приседания и тяги добавляются тренировки по развитию сердечно-сосудистой системы, состоящие из многоповторных приседаний, бега на короткие дистанции и т. д.

    По фулбоди занимаются, чередуя тренировочный день, и день отдыха.

    Сплит «верх-низ»

    Тренировки в стиле «верх в один день, низ – в один день» типичны для американского бодибилдинга и пауэрлифтинга. Вокруг них выросли целые методические системы, например, «Куб» Брендона Лилли.

    Брендон Лилли – создатель системы “Куб”

    • Хорошо работают, если нужна масса и сила;
    • Позволяют развить нужные навыки для любого вида спорта;
    • Не дают слишком высокой нагрузки на суставы и связки;
    • Подходят мужчинам и женщинам.

    Для натурального атлета этот вариант почти не имеет минусов. Но в профессиональном бодибилдинге он часто не дает нужного результата, так как просто нет времени на развитие отдельных мелких групп мышц.

    Трехдневный сплит «жим-тяга-ноги»

    Сплит «жим-тяга-ноги» обычно выглядит так:

    День 1 – жим лежа, упражнения на трицепс, переднюю дельту, и среднюю дельту;

    День 2 – становая тяга, или тяга в наклоне, проработка спины, бицепсов, задних дельт;

    День 3 – приседание и тренировка ног

    Можно выполнять упражнения на плечи в день ног, если нет сил в другие тренировки добавлять их в план, либо они отстают. Вариант удобен для большинства занимающихся, так как предполагает посещение зала 3 раза в неделю.

    Четырехдневный сплит

    Эта схема для тех, кто желает качать руки в отдельный день, и видит в этом смысл. Обычно «разбивка» такая:

    День 1 – грудь и средняя дельта;

    День 2 – спина и задняя дельта;

    День 3 – ноги и передняя дельта;

    День 4 – бицепсы и трицепсы (они являются антагонистами, это и дало название сплиту)

    Подходит вариант для более продвинутых атлетов, которые считают, что у них отстают руки.

    Пятидневный сплит

    Пятидневный сплит – для продвинутых спортсменов, чей уровень близок к профессиональному. Тренируются в этом стиле часто и худеющие, просто потому, что это мотивирует ходить в зал каждый день, и тратить больше калорий.

    День 1 – ноги, передняя поверхность бедер;

    День 2 – грудь и трицепс;

    День 3 – спина и бицепс;

    День 4 – ноги, задняя поверхность бедер;

    День 5 – дельты.

    Возможны и другие варианты пятидневного сплита, в зависимости от отстающих групп мышц.

    Заключение

    Критерии построения тренировочной программы- это подготовка человека, его мышечное развитие, цели, и скорость восстановления. Частые тренировки нецелесообразны, если человек не может поддерживать режим дня, и правильно питаться. Большинству людей с обычной работой и обязанностями подходит трехдневный сплит. Но могут быть исключения. Есть профессиональные атлеты, занимающиеся только по фулбоди, и новички с пятидневным сплитом. Сомневающиеся относительно планирования могут воспользоваться услугами тренера, или начать тщательно отслеживать свое состояние, и воплощать свои особенности в программе самостоятельно.

    Источники:

    http://tutknow.ru/bodyfitness/9810-kak-sovmeschat-myshechnye-gruppy-na-trenirovke.html
    http://gymlex.com/faq/kakie-myshcy-sovmeschat-vmeste.html
    http://muskul.pro/t/kakie-gruppy-myshts-luchshe-sovmeshhat

    Какие группы мышц следует тренировать вместе | Mysportonline.ru

    Суть господствующей сегодня в бодибилдинге системы сплита состоит в том, что на одной тренировке можно нагружать 2 от силы 3 мышцы. Но в каком сочетании? Многие специалисты считают, что наилучшим образом сочетаются на одной тренировке одна большая мышечная группа с ассистирующей ей малой мышцей. К примеру, трицепс и грудь. Причем в основном нагрузке следует подвергать большую мускульную группу. Если тренировку начать с ассистирующей мышцы, то ее утомление негативно отразится на результативности основной мышечной группы. Так же не желательно тренировать одновременно мышцы спины и груди как две большие мускульные группы. 

    Где же истина? 
    Многочисленные эксперименты с участием силовиков выявили, что оказывается спину и грудь можно тренировать совместно, только грудь следует нагружать первой. Что касается бицепса и трицепса, то вне зависимости от порядка тренинга они не демонстрируют потерю силы. А вот дельты ведут себя, как и мышцы груди. То есть выходит, что дельты и грудь сочетать нельзя. 

    Порядок проработки мускульных групп. 
    Наибольшее количество анаболических гормонов высвобождается во время упражнений с умеренным весом при множестве сетов, состоящих из 10-12 повторений, и периодом отдыха 1,5 минуты. Речь идет о таких гормонах, как тестостерон, гормон роста и схожий с инсулином фактор роста-1. Так вот, при тренировке больших групп мышц осуществляется более значительный синтез гормонов, нежели при тренинге, таких как трицепс или бицепс, то есть малых. А что случится, если одновременно прорабатывать и те и другие? Распространится ли эффект от первых на последние? Оказывается, именно так все и происходит. Это нашло подтверждение во время многочисленных исследований. К примеру, две группы ранее не тренировавшихся мужчин выполняли упражнения: первая работала с бицепсами, вторая — после тренировки ног с теми же бицепсами. Так вот во второй группе был выявлен гораздо больший выброс анаболических гормонов. 

    Известный еще контрлатеральный эффект состоит в том, что при тренировке одной стороны тела стимулируется и вторая, ей симметричная. То есть, тренируя одну руку можно в незначительной степени увеличивать массу и силу второй. Причина — возбуждение нервов, которые стимулируют обе стороны тела. 
    Популярным у бодибилдеров является принцип «тяга-жим», при котором в один день вы тренируете после спины трицепсы, а в другой – бицепсы после груди. В любом случае первыми надо тренировать большие мускульные группы. Причин тут две: 
    Они нуждаются в больших затратах энергии; 
    Нагружая их, мы обеспечиваем максимум анаболиков. 

    Разбиваем основные мышечные группы по дням: 
    1 – грудь; 
    2 – ноги; 
    3 – спина 
    Затем мы дополняем их тренинг второстепенными мышцами. 
    1 день. Выполняем базовые упражнения на грудь, при этом задействована и второстепенная мышца – трехглавая. Поэтому после нагрузок на грудь добиваем трицепс. Второстепенная мышца получает отличную загруженность и время для восстановления, потому что другие дни ни грудь, ни трицепс не будут задействованы и пройдут полное восстановление, что очень важно при наборе мышечной массы. 
    2 день. Ноги и после плечи. 
    3 день. Спина и бицепс. Бицепс помогает как сгибатель уже в первом упражнении на спину. После спины выполняем тренинг бицепса. 

    Другая схема распределения: 
    Тренировка 1. Прокачка ног, дополнительно загружаются столбы спины и поясница. Если еще есть силы, нагрузите бицепс. 
    Тренировка 2. Нагрузка на грудь, трицепс, дельты передние, пресс. При прокачке груди будет задействован трицепс вместе с передними дельтами. Прокачав передние дельты, трицепс надо лишь добить. Пресс в конце. 
    Тренировка 3. Полностью спина. Верх спины невозможно прокачать без привлечения бицепса. 
    Тренировка 4. Плечи задействуют трицепс. Не забывайте про пресс, шею и трапеции. 

    Пример сплита 
    Понедельник: грудь, плечи, трехглавая мышца плеча; 
    Среда – спина, двуглавая мышца плеча, предплечья; 
    Пятница – бедра, ягодицы, мышцы голени 
    Пресс качаем каждую тренировку: 
    по понедельникам – верх; 
    по средам – косые мышцы живота; 
    по пятницам – низ пресса. 

    Заключение 
    Единой системы просто нет. Многое зависит от особенностей каждого индивидуума, и за какое время происходит восстановление. 
    Причин для грамотного сочетания мышечных групп – две, дней для реализации такого тренинга -3-4, а условие только одно – посещение тренировочного зала минимум 2-3 раза в неделю, что уже станет гарантией эффективной и грамотной проработки всех мышечных групп.

    Подписывайтесь на мой канал, впереди еще много интересного!

    Когда лучше тренировать трапеции: в день спины или плеч?

    Трапециевидные мышцы участвуют в упражнениях для плеч и спины. Узнайте, как и когда тренировать каждую их часть, чтобы добиться максимального развития.

    Автор: Билл Гейгер

    Если только вы не новичок, ваши тренировки разделены по частям тела, и вы используете серию упражнений, чтобы проработать конкретную мышечную группу под разными углами и форсировать максимальный рост. Такой подход в большинстве случаев оправдан, но трапециевидные мышцы, распластанные по всей поверхности спины, порождают определенную проблему.

    Кто-нибудь говорил вам: «У меня сегодня день трапеций»? Я никогда таких не встречал. Что отличает трапеции от остальных мышц, так это то, что мышца в форме бриллианта состоит из трех регионов — верхнего, среднего и нижнего — и каждый выполняет определенную функцию. И хотя трапеция — это одна мышца, спортсмены часто прорабатывают ее части в разные тренировочные дни. Это немного сбивает с толку, так что давайте выделим функции каждой части трапециевидной мышцы, чтобы окончательно определиться, когда их все-таки лучше тренировать.

    В разведении рук в стороны на средние дельты плечи поднимаются, а это значит, что трапеции участвуют в движении

    Когда вы представляете себе накачанные трапеции, в голову, скорее всего, приходит верхняя их часть между шеей и плечами. Когда вы поднимаете лопатки или вращаете их вверх, верхние пучки включаются по-максимуму. Аналогичное движение мы делаем, когда пожимаем плечами в обычной жизни. Помимо шраг, многие упражнения на дельты, особенно разведения в стороны и тяга к подбородку, нагружают верхнюю часть трапеций почти напрямую, так что есть смысл заканчивать тренировки односуставными шрагами. Следовательно, верхние пучки трапециевидных мышц лучше тренировать в день плеч.

    Средний отдел трапеции, главным образом, стягивает лопатки друг к другу, и называется это движение ретракция. Вы имитируете ретракцию в зале, когда выполняете тягу на нижнем блоке сидя. (И если вы не сводите лопатки вместе во время тягового движения, вы обкрадываете среднюю часть трапеций!) Тяги обычно делают в день спины, но движение нагружает и другие мышечные группы. Чтобы лучше изолировать трапециевидные мышцы, выполняйте тягу сидя, не сгибая локти (тяга нижнего блока прямыми руками). Просто сводите лопатки вместе и разводите их, так чтобы амплитуда движения была минимальной. Получается, что средние трапеции лучше тренировать в день спины.

    Первые 10-15 сантиметров тяги на верхнем блоке активируют нижние трапеции

    Чтобы окончательно все запутать, обратим взоры на нижние трапеции. Их функция — вращение лопаток вниз, а это движение происходит в начальной фазе тяги верхнего блока, еще одного упражнения для спины. Следовательно, вы можете успешно изолировать нижние трапеции, выполняя тягу на верхнем блоке на 10-15 сантиметров с прямыми руками. Нижние волокна также участвуют в подъеме на дельты перед собой на наклонной скамье лицом вниз, а это уже упражнение для плеч! Даже в рывке, олимпийском упражнении, в котором участвует все тело, работают нижние волокна трапеций.

    Итак, где место трапеций в вашем тренировочном сплите? Вариативно. Если вы не прорабатываете спину и плечи в один день, скорее всего, вы будете нагружать разные отделы мышцы на тренировках для разных целевых групп, и это идеальный вариант. Просто помните, что для полноценной тренировки нужно использовать всю палитру движений. Слишком часто спортсмены ограничиваются шрагами. Это неправильно, и теперь вы знаете, почему!

    Читайте также

    Какие группы мышц тренировать вместе девушкам. Какие мышцы тренировать за одну тренировку

    Привет покорителям железных рубежей! В одном из последних сюжетов мы говорили о вашей мечте и постановке цели для того, чтоб мотивировать себя на ее достижение. Однако, как говориться, мечты – это планы в уме, а планы – это мечты на бумаге.. Тема нашего сегодняшнего сюжета – это, как вы уже догадались, тренировочные планы, или программы. Я достаточно педантичный человек и верю, что даже самый дурацкий замысел можно выполнить мастерски, если долго и систематически следовать плану. Кстати, тут вы сразу же услышали и первый совет — для отдачи от любого плана нужно время и последовательность. Часто новички прыгают с одного тренировочного сплита на другой, просто не давая своему телу прочувствовать преимущества конкретной тренировочной программы. Если вы не хотите постоянно топтаться на месте, то будьте готовы выбрать план следовать ему многие месяцы, иначе вы просто не поймете, эффективен ли он вообще для вас или нет. Ок. Как же составляют тренировочную программу персональные тренеры?

    Персональный Тренер и Тренировочная Программа.

    Каждый человек, когда начинает заниматься, хочет верить в некие секретные методы и программы тренировок, так же, как волшебные протеины и прочие таблетки. Это так называемый синдром «волшебной палочки». С этой целью новички идут к персональным тренерам и просят написать им «волшебную» Персональную программу, от которой их мышцы просто разорвет и они станут похожи на Арнольда Шварценеггера через пол года. К счастью, чудеса бывают только в сказках, а результат всегда требует тяжелой работы над собой. Так что готовьтесь потеть, друзья.

    Чаще всего Персональный Тренер всем новичкам пишет одну и ту же программу с небольшими вариациями на свой вкус. К примеру, если подойдет девушка или худой подросток – даст больше повторений в каждом подходе и выкинет некоторые опасные упражнения из тренировочного сплита. В целом же, если у тренера сложились определенные представления о тренинге, он будет всем своим клиентам давать практически одно и то же. Плохо ли это? Чаще всего, если тренер не полный дуб, то это не так уж и плохо. Потому что идеально подходящую персонально для вас программу составить с первого раза НЕ ВОЗМОЖНО! Это все равно, что во время боевых действий из миномета без пристрелки и коррекции сразу попасть в далекую цель. Наводчик обычно делает первый выстрел, смотрит на результат, делает коррекцию, второй выстрел, и вот уже с третьего он, зная все особенности, может рассчитывать на прямое попадание. Так же и в составлении тренировочной программы. Пока мы не знаем, как наше тело будет реагировать на ту или иную нагрузку, мы не можем выбрать самый лучший способ прогрессии этой нагрузки.

    Главное, чтоб тренер не начал писать уж полную отсебятину. Ну, к примеру, заставил бы вас делать 20-30 повторений или отдыхать 3 минуты, в то время как ваша цель – набор мышечной массы. Короче, если коротко, основная задача при составлении первой тренировочной программы – это выбрать примерно верное направление. И в последствии это направление корректировать для лучшей точности и результативности. Поэтому любая программа лучше, чем вообще ее отсутствие. Что же касается того, как выбирать «верные направления», мы сейчас постараемся разобраться.

    Частота тренировок

    Как часто тренировать конкретную мышечную группу и как часто ходить на тренировки вообще? Слушайте основные моменты, друзья! У нас есть крупные мышцы (Спина, Ноги, Грудь) и мелкие мышцы (Дельты, Бицепс, Трицепс, Икры) . Первые восстанавливаются дольше, чем вторые из за своего размера. Иначе говоря, для крупных групп мышц может понадобится на один день больше отдыха, чем для мелких. Следующий важный момент – это объем приложенной работы во время тренировки. Чем тяжелее и дольше вы тренировали мышцу, тем более длительный отдых ей нужен. И последний важный момент – это ваша тренированность. Чем дольше вы тренируетесь, тем больше с одной стороны вы адаптированы переносить нагрузку и быстрее восстанавливаться. С другой стороны, чем больше у вас становятся ваши мышцы, тем больше времени им нужно для восстановления. Вот такой парадоксик. Причем, рост восстановительных возможностей будет отставать. К примеру, вы увеличили вашу руку с 30 см до 50 см…. Восстановительные возможности увеличились на 50%, а количество необходимого отдыха может увеличится на 200%. Эту информацию обычно не дают новичкам. Но я хочу, чтоб вы развивались быстро и уже сейчас задумывались бы над будущими достижениями, поэтому говорю о тех «подводных камнях», с которыми вы столкнетесь в своих будущих плаваниях.

    Вообще, у каждого тренера существует свои представления о количестве отдыха между тренировками. Я, лично, считаю, что для новичков достаточно 2-4 суток между тренировками одной мышечной группы. Это не очень длительный отдых и я часто рекомендую его новичкам по двум причинам: во-первых, их мышцы достаточно «маленькие» и восстановятся быстрее, чем у профессионального бодибилдера, а во-вторых, новички сокращают свои мышцы с низким КПД (связь мозг-мышцы у новичка еще работает не настолько эффективно, как у опытного атлета). Поэтому начинающим можно тренироваться достаточно часто с низкой или умеренной интенсивностью для того, чтоб привыкнуть к подобным нагрузкам и научить мышцы сокращаться с более высоким КПД.

    Продолжительность Тренировки и Отдых между Подходами

    Продолжительность тренировки чаще всего должна укладываться в 1 час, если вы тренируетесь натурально (т.е. без использования ААС). Почему? Просто потому что тренировка – это сильный стресс для вашего тела и гормональной системы. Тренировка – это всегда выброс кортизола и других катаболиков. Если затянуть время их высвобождения, то восстановление после этого стресса замедлиться в связи с замедлением выброса анаболических гормонов. В частности тестостерона. Естественно, если вы колите искусственные гормоны, то можете тренироваться дольше, т.к. на вас это не распространяется.

    Отдых между подходами желательно делать в пределах одной минуты. Потому что подобный режим позволит вам сделать гораздо больший объем работы на тренировке, чем если бы вы отдыхали по 3 минуты между подходами. Именно в этом суть бодибилдинга, если сравнивать его с пауэрлифтингом. Мышцы больше всего растут от объемной работы с средними весами. А сила растет от низкообъемной работы с тяжелыми весами.

    В общем, составляя тренировочную программу, на каждый подход прикидывайте 90-120 секунд (30 сек – подход и 60-90сек – отдых). В итоге, если тренировка у нас длиться 60 минут, вы можете в нее впихнуть максимум 30 подходов. Что достаточно много и является верхней границей. Обычно достаточно гораздо меньшего количества подходов.

    Сплит и порядок тренировок различных мышц

    Можно тренировать все тело за раз (как делали наши дедушки или как, бывает, рекомендуют новичкам), а можно расщеплять мышечные группы для тренировки в различные дни. Последний способ называется сплит (расщеплять) и считается более прогрессивным, потому что позволяет более сконцентрировано работать над каждой мышцей. С другой стороны, чем больше вы расщепляете ваше тело на различные дни, тем меньше у вас остается полных дней отдыха для восстановления. А рост мышц идет именно в дни отдыха. Это не так уж важно для больших парней, т.к. их мышцы могут восстанавливаться очень долго – 7-10 дней, но это может вредить начинающим, которым нужно тренировать группы чаще. Поэтому вот вам первое правило тренировочного сплита: Чем более вы зеленый, тем меньше расщепляйте тело по разным дням. Чем больше ваши мышцы, тем больше тренировочных дней вы можете делать в вашем комплексе.

    Привет всем, уважаемые читатели моего блога. Сегодня я рассказываю о том, какие мышцы можно и нужно тренировать вместе, что бы активировать выброс анаболических гормонов и ускорить рост наших мышц.

    На сегодняшний день в бодибилдинге есть прочное убеждение о том, что на одной тренировке можно тренировать не менее 2-3 мышечных групп. Профессионалы, кстати, бомбятся вообще по одной группе в день. Но мы не профи, мы будем работать в другом ритме.

    В следующей статье я расскажу вам, как возможно относительно быстро похудеть и сбросить вес. Тема будет особенно интересна девушкам, в преддверии пляжного сезона. Да и мужикам будет полезно подсушиться, у кого проблемы с этим. Не забудьте подписаться на выход новых статей моего блога, что бы не пропустить этот пост.

    Когда проводили исследования, то выяснили, что максимальный эффект достигается в двух случаях.

    1. Когда вы тренируете только большие мышечные группы. Спину, грудь и ноги.
    2. Когда вы совмещаете тренировку большой мышечной группы с тренировкой малой, сопутствующей мышечной группы. Например, совмещаете тренировку груди с тренировкой трицепса, тренировка плеч, сочетается с тренингом ног, а тренировка спины очень хорошо сочетается с тренировкой бицепса.

    В первом случае вы просто интенсивно тренируете большие мышечные группы, которые как локомотив тянут за собой всё тело. Упражнения тут вы выбираете тяжёлые, базовые. Жимы, тяги. Приседы и так далее. Тяжёлая база по полной программе нагружает и плечи и мышцы рук, так как все жимы и тяги выполняются и с их помощью то же. Поэтому развивая грудь, вы обязательно будет иметь и сильные руки то же.

    Во втором случае мы бомбим сопутствующую мышечную группу дополнительными упражнениями и вызываем дополнительный её рост. Главное правило тут тренировать вначале большую мышечную группу, а потом мышцу сопутствующую. Например, вначале работаем грудь, потом трицепс. Мышцы рук в этом случае нам нужны свежие, что бы прокачать большую и более важную для нас мышечную группу груди. А трицепс мы уже дорабатываем после.

    Такая же схема применима к тренировкам спины-бицепса, и ног-плеч. Вначале работаем большую группу, потом сопутствующую.

    Количество подходов и повторов выбирайте сами в зависимости от вашего телосложения и обмена веществ. Если вы мезоморф, то 3-4 подхода с 10-12 повторами для вас будет самое оно. А если вы, к примеру, эктоморф, тогда ваша схема – 2 подхода с 8-10 повторами.

    Можно привести в качестве примера такую схему тренировки.

    Понедельник

    Грудь

    Трицепс

    Пресс

    Среда

    Ноги

    Плечи

    Пресс

    Пятница

    Спина

    Бицепс

    Пресс

    Как видите, пресс мы тренируем каждую тренировку. На него достаточно будет 2 упражнений за тренировку. Прямые+нижние, или прямые+ косые, к примеру.

    Если же вы имеете возможность тренироваться только два раза в неделю, тогда логичнее будет тренировать только большие мышцы плюс какую-то одну малую. Схема может выглядеть так.

    Понедельник

    Грудь

    Спина

    Пресс

    Четверг

    Ноги

    Плечи

    Пресс

    Упражнения советую выбирать базовые, в особенности, если у вас цель накачать массу. Количество повторов и сетов – выбирайте сами исходя из вашего телосложения, вашей подготовленности и возможности вашего кошелька покупать качественный «спортфуд».

    Опять же повторяюсь. Не забывайте о питании и грамотном отдыхе. Любая тренировка должна компенсироваться и поддерживаться питанием и отдыхом. Чем жёстче тренировка, тем обильнее и качественнее должен быть отдых и питание растренированных мышц. Вносите изменения в свой тренировочный режим – это, пожалуй, самое важное для предотвращения застоя в результатах и прогрессе.

    В качестве учебного пособия для самостоятельных тренировок могу порекомендовать Бесплатный курс о наборе массы . Посмотрев 5 видео уроков вы узнаете, как избежать основных ошибок, при наборе массы, как накачать красивую грудь и составить свою программу питания для набора мышечной массы в процессе тренировок.

    Для тех, кто хочет поучить более продвинутые методики тренинга, рекомендую ознакомится с платным тренингом «Базовый курс на массу» Автор Владимир Молодов, бодибилдер с 11-ти летним стажем. В курсе ещё больше фишек и приёмов, которые помогут вам набрать массу всего за 90 дней. И отзывы это подтверждают.

    А на сегодня у меня всё, до встречи в следующей статье про похудение! Берегите себя.

    Анекдот дня.

    Что это у тебя с лицом?

    Вчера ходил в тренажерный зал, хотел себя в форму привести…

    При чем тут тренажерный зал, я тебя про лицо спрашиваю. Что с лицом???

    Так в этом зале вчера один качок гирю уронил…

    Нет! Себе на ногу !

    А лицо?

    А мое лицо решило над этим посмеяться…

    P/S Не нужно сдерживать эмоции, идущие от сердца! Выплесни их в комментариях! Будет интересно узнать, что ты думаешь по поводу прочитанного!
    Не забудьте подписаться на обновление блога , что бы первым получать свежие статьи!

    Суть господствующей сегодня в бодибилдинге системы сплита состоит в том, что на одной тренировке можно нагружать 2 от силы 3 мышцы. Но в каком сочетании?

    Многие специалисты считают, что наилучшим образом сочетаются на одной тренировке одна большая мышечная группа с ассистирующей ей малой мышцей. К примеру, трицепс и грудь. Причем в основном нагрузке следует подвергать большую мускульную группу.

    Если тренировку начать с ассистирующей мышцы, то ее утомление негативно отразится на результативности основной мышечной группы. Так же не желательно тренировать одновременно мышцы спины и груди как две большие мускульные группы.

    Где же истина?

    Многочисленные эксперименты с участием силовиков выявили, что оказывается спину и грудь можно тренировать совместно, только грудь следует нагружать первой. Что касается бицепса и трицепса, то вне зависимости от порядка тренинга они не демонстрируют потерю силы. А вот дельты ведут себя, как и мышцы груди. То есть выходит, что дельты и грудь сочетать нельзя.

    Порядок проработки мускульных групп

    Наибольшее количество анаболических гормонов высвобождается во время упражнений с умеренным весом при множестве сетов, состоящих из 10-12 повторений, и периодом отдыха 1,5 минуты. Речь идет о таких гормонах, как тестостерон, гормон роста и схожий с инсулином фактор роста-1.

    Так вот, при тренировке больших групп мышц осуществляется более значительный синтез гормонов, нежели при тренинге, таких как трицепс или бицепс, то есть малых. А что случится, если одновременно прорабатывать и те и другие? Распространится ли эффект от первых на последние?

    Оказывается, именно так все и происходит. Это нашло подтверждение во время многочисленных исследований. К примеру, две группы ранее не тренировавшихся мужчин выполняли упражнения: первая работала с бицепсами, вторая — после тренировки ног с теми же бицепсами. Так вот во второй группе был выявлен гораздо больший выброс анаболических гормонов.

    Известный еще контрлатеральный эффект состоит в том, что при тренировке одной стороны тела стимулируется и вторая, ей симметричная. То есть, тренируя одну руку можно в незначительной степени увеличивать массу и силу второй. Причина — возбуждение нервов, которые стимулируют обе стороны тела.

    Популярным у бодибилдеров является принцип «тяга-жим», при котором в один день вы тренируете после спины трицепсы, а в другой – бицепсы после груди. В любом случае первыми надо тренировать большие мускульные группы.

    Причин тут две:

    • Они нуждаются в больших затратах энергии;
    • Нагружая их, мы обеспечиваем максимум анаболиков.

    Разбиваем основные мышечные группы по дням:
    1 – грудь;
    2 – ноги;
    3 – спина

    Затем мы дополняем их тренинг второстепенными мышцами.

    1 день. Выполняем базовые упражнения на грудь, при этом задействована и второстепенная мышца – трехглавая. Поэтому после нагрузок на грудь добиваем трицепс. Второстепенная мышца получает отличную загруженность и время для восстановления, потому что другие дни ни грудь, ни трицепс не будут задействованы и пройдут полное восстановление, что очень важно при наборе мышечной массы.

    2 день. Ноги и после плечи.

    3 день. Спина и бицепс. Бицепс помогает как сгибатель уже в первом упражнении на спину. После спины выполняем тренинг бицепса.

    Другая схема распределения:

    Тренировка 1. Прокачка ног, дополнительно загружаются столбы спины и поясница. Если еще есть силы, нагрузите бицепс.

    Тренировка 2. Нагрузка на грудь, трицепс, дельты передние, пресс. При прокачке груди будет задействован трицепс вместе с передними дельтами. Прокачав передние дельты, трицепс надо лишь добить. Пресс в конце.

    Тренировка 3. Полностью спина. Верх спины невозможно прокачать без привлечения бицепса.

    Тренировка 4. Плечи задействуют трицепс. Не забывайте про пресс, шею и трапеции.

    Пример сплита

    • Понедельник: грудь, плечи, трехглавая мышца плеча;
    • Среда – спина, двуглавая мышца плеча, предплечья;
    • Пятница – бедра, ягодицы, мышцы голени
    • Пресс качаем каждую тренировку:
    • по понедельникам – верх;
    • по средам – косые мышцы живота;
    • по пятницам – низ пресса.

    Заключение

    Единой системы просто нет. Многое зависит от особенностей каждого индивидуума, и за какое время происходит восстановление.

    Причин для грамотного сочетания мышечных групп – две, дней для реализации такого тренинга -3-4, а условие только одно – посещение тренировочного зала минимум 2-3 раза в неделю, что уже станет гарантией эффективной и грамотной проработки всех мышечных групп.

    На фото Арнольд Шварценеггер

    Вконтакте

    Одноклассники

    Любой мужчина, который хочет, чтобы его уважали в обществе не только за его ум, часто задумывается о своих физических данных. Однако одно дело думать об этом, а другое дело начать.

    Эта статья самым непосредственным образом расскажет о том, как именно нужно действовать, чтобы Ваш прогресс в фитнесе поразил окружающих.

    Где заниматься — дома или в зале?

    Часто у людей, далеких от спорта, появляется вопрос: где лучше тренироваться, в спортивном зале или дома? Ответ зависит от Ваших целей. Если задачей является приобретение объемной гармоничной мускулатуры, то путь один — фитнес-центр или спортивный зал.

    А если Вы хотите просто немного изменить свое телосложение, то подойдут и домашние тренировки, но без дополнительного веса прогресс быстро остановится, а покупка домой гантели и штанги обойдется едва ли дешевле, чем абонемент в зал.

    Поэтому после занятий дома для того, чтобы продолжать расти и развиваться в любом случае нужно идти в спортивный клуб. А есть ли тогда смысл терять время дома, когда можно будет заниматься в специально оборудованном центре с куда большим комфортом и прогрессом?

    Как начать тренироваться?

    Для начала тренировок следует точно поставить себе цель, ведь правильно подобранная мотивация — это уже половина победы.

    Большинство читателей не нужно мотивировать дополнительно, потому что главная мотивация — отражение в зеркале. И если кто-то скажет Вам, что не хотел бы набрать несколько килограмм качественной мышечной массы, то, поверьте, он как минимум лукавит.

    Занятия следует начать с обследования у доктора, если Вы этого не делали раньше. Ведь, возможно, некоторые упражнения следует исключить из тренировочной программы (о составлении программы расскажем ниже).

    Такое иногда случается, например, при травмах спины или шеи, но даже если у Вас есть такие проблемы, то не расстраивайтесь, упражнений сейчас существует огромное количество и существуют разные способы работы на определенные мышцы, которые можно использовать даже при травмах.

    Когда первый этап пройден, пора переходить к покупке абонемента. Тут не будет идеального совета, но все же лучше посещать спортивный зал, который находится рядом с домом, работой или учебой, ведь после тренировки очень важен отдых.

    На данный момент сейчас такое обилие фитнес-центров, что любой читатель найдет себе зал по душе. Единственный практический совет, который можно дать: не скупитесь на своем здоровье. Ведь за редким исключением цена прямо пропорциональна качеству предоставляемых услуг.

    Согласитесь, поплавать в теплом бассейне и посетить после тренировки не только очень приятно, но еще и полезно.

    Разминка

    Каким видом спорта Вы бы не занимались, следует запомнить одну очень важную вещь, без которой тренировки не только не принесут результата, но еще и могут навредить здоровью. Речь идет о разминке.

    Ведь тяжелые веса, без которых немыслима тренировка для набора мышечной массы, разрушающе повлияют на неразогретые мышцы и суставы.

    Согласитесь, обидно будет получить серьезную травму из-за того, что захотелось сэкономить 5-10 минут на простых разогревающих упражнениях.

    Как именно разминаться знают все еще со школьных уроков физкультуры: простые движения, начинающиеся с верхней части тела.

    Также перед каждым упражнением следует сделать несколько подходов с небольшими весами, постепенно повышая вес до рабочего. Это прекрасно разогреет мышцы и суставы и защитит от травм.

    Программа тренировок как правильно качать мышцы

    Вот Вы купили абонемент, приступили к тренировкам. Главной ошибкой начинающих на этом пункте является совершенно непродуманная программа тренировок. На данный момент существует множество программ как правильно качать мышцы тела и каждая из них имеет свои преимущества.

    На начальном этапе больше всего подойдет такая схема: 1-ая тренировка: ноги, плечи, 2-ая: грудь, трицпсы, 3-я спина, бицепсы.

    Бросается в глаза отсутствие конкретных упражнений, написаны только группы мышц, которые следует нагружать в каждую тренировку, и это неспроста.

    Существует огромнейшее количество упражнений на каждую группу мышц, но на начальном этапе свое внимание следует обратить на несколько главных упражнений, благодаря которым с Вашим телом при правильном выполнении и питании будут происходить настоящие чудеса.

    И так, начать следует со «святой» для фитнесистов и бодибилдеров тройки упражнений. А именно: жим лежа, присед со штангой на плечах и становая тяга. Если у Вас есть проблемы со спиной, то от двух последних упражнений придется отказаться, либо выполнять их очень аккуратно.

    Возникает вопрос, почему же именно эти три упражнения так влияют на рост мускулатуры? Ведь те же мышцы можно нагрузить в тренажерах. Ответ уже давно дали ученые, занимающиеся биохимией.

    В этих упражнениях включаются в работу почти все группы мышц, реакция организма не заставляет себя долго ждать. Начинает выделяться огромное количество тестостерона, гормона, который делает мужчину мужчиной, который в разы ускоряет синтез белка в мышцах.

    Конечно, не стоит забывать и об остальных упражнениях, но большую часть прогресса дадут именно базовые упражнения.

    Остальные упражнения, которые будут хороши как на начальном этапе, так и на протяжении всех тренировок:

      жим штанги или гантелей над головой — плечи

      подъем штанги или гантелей на бицепс

      подтягивания, тяга блоков — спина

      жим узким хватом — трицепс

    От правильной техники выполнения упражнений зависит абсолютно все в ваших тренировках. А именно прогресс и отсутствие травм. Что следует запомнить раз и на всегда, так это то, что спину во всех упражнениях следует держать прямой.

    Второе, что нужно помнить — никогда не позволяйте себе читерить, если только это не последний подход . Что под этим подразумевается? Например, раскачивание при подъеме штанги на бицепс.

    В идеале это упражнение стоит делать стоя у стены, прижавшись лопатками и тазом. В этот список можно включить и разведение локтей в стороны при жиме узким хватом.

    Читинг — неправильные движения во время упражнения, которые заставляют работать совершенно не те мышцы, которые нужно, но позволяют поднимать большие веса.

    Использовать его целесообразно в конце тренировки, чтобы окончательно «добить» определенные группы мышц.

    Сколько тренироваться в неделю


    Часто новичок жалуется на отсутствие прогресса. А в ответ на вопрос «Как часто ты тренируешься» спокойно говорит «Каждый день». И все становится понятно.

    Всем давно известно, что мышцы растут не во время тренировок, а во время отдыха, особенно сна. Поэтому если не давать отдыха своему организму, то вместо роста мышц (анаболизма) начинается совершенно иной процесс — катаболизм.

    Так называется разрушение белка в мышцах, это приводит только к потере объемов, которые Вы набирали так старательно.

    После тренировки организму нужно давать отдых. Как минимум это будет 1 день, но оптимальный вариант — 2 дня отдыха между каждой тренировкой. Так организм будет успевать восстанавливаться в полной мере, и занимающийся будет подходить к каждой тренировке в идеальной форме.

    Мечтаете о развитой мускулатуре? В данной статье рассматриваются упражнения для набора мышечной массы.

    Сейчас в бодибилдинге основной является сплит. Его назначение — на одном занятии нагрузка идет сразу на несколько мышц. Но как их комбинировать? Опытным путем выяснилось, что лучше сочетать за 1 подход большие группы мышц с помогающими им малыми.

    Например, грудную и трицепс. Главная нагрузка при этом должна приходиться на большие. Если упражнение начинать с помогающей мышцы, то она скоро устанет, и это негативно скажется на всех мышцах.

    Советуют одновременно нагружать спинные и грудные мышцы, которые образуют две группы. Проведя тренинги, оказалось, что грудь и спину можно нагружать вместе в случае, если грудь тренировать первую.

    При тренингах бицепса и трицепса порядок нагрузки не имеет значения, так как их функции в сочетаниях сохраняются. Но дельты не сочетают с грудными мышцами, так как они ведут себя похоже.

    Интереснейшие статьи для вашей тренировки:

    Как прорабатывать


    Большое число гормонов анаболиков выделяется на занятиях со средним весом в 10-12 повторов и паузами полторы минуты. Большие мышечные группы на занятиях дают более мощный гормональный выброс в сравнении с тренингами малых. А что если их также сочетать с большими группами?

    Провели исследование: взяли две группы мужчин без подготовки. Первая нагружала бицепсы, а вторая тренировала ноги, но вместе с бицепсами. В этой второй группе был более мощный уровень гормонов анаболиков.

    Есть еще один эффект и утверждение, что при нагрузке правой стороны тела проходит стимуляция левой, и наоборот. То есть, сила и вес мышц с другой стороны возрастает из-за возбуждения нервов.

    Эффективный способ тренировки у бодибилдеров — система тяга-жим. В одно занятие проводят тренинг сначала спины, а затем трицепсов. А на другое — грудь, а затем бицепсы. В каждом случае нагружают первыми большие группы мышц.

    Считается, что они требуют большого расхода энергии. Но при их нагрузках идет максимальный выброс гормонов. Тренинги разделяют по дням. Но все занятия ещё дополняют нагрузками на второстепенные мускулы.

    Первое занятие — упражнения на грудь, и дополнительно нагружают еще и трехглавую мышцу, а затем тренинг трицепса. Трехглавая мышца хорошо нагружается, и затем ей дается большой период отдыха, так как в другие дни она и грудь не тренируются. Это важно, чтобы нарастить массу.


    Второе — сначала ноги, затем мышцы плеч.

    Единых систем не создано, ещё все зависит от конституции спортсмена, и за какой период он сможет восстановиться. Но грамотный подход учитывает — за одно занятие нагружают 2 группы мышц, а дня 3-4 число тренировок — 2-3 раза в месяц. Это обеспечит эффективную проработку всех мышечных групп.

    Что произойдет, если 2 недели не тренироваться

    Пропуски занятий чреваты потерей мышцами набранной силы и объема. Если появилась у спортсмена потребность хорошо поспать, то это не отразится на силе даже в случае пропуска занятия. Особенно если при этом усиленно питаться и отдыхать. Ущерба, как говорят спортсмены, не будет.

    Но перемены начинают ощущаться уже на третий день. Даже такая незначительная пауза ведет к уменьшению мышц. Калории сжигаются медленнее, и сила теряется.

    Глядя в зеркало, это поначалу не видно. Но волокна мышц, которые обычно дольше сокращаются, а также тренируемых на выносливость, начинают скоро уставать. Но пропуск одного занятия не играет особой роли.

    Группы мышц, нагруженных и в обычной жизни (бицепсы бедер), не теряют силу так быстро, как те, которые редко включаются (пресс). Тренеры также утверждают, что нужно будет вдвое больше времени, чем пропущено, чтобы всё восполнить.

    Так, при отдыхе 2 недели нужен будет месяц занятий, чтобы обрести прежний уровень.

    Но считается, что отдых не является негативным, если его долго не затягивать. К тому же, если продолжить потреблять много белка, можно стать даже еще сильнее.


    Многие задают вопрос, почему при отдыхе хочется есть? При пропусках занятий есть много шансов потреблять больше жиров и углеводов, а не здоровых продуктов. Это объясняется тем, что уже нет положительного влияния занятий на общее настроение и энергетику.

    При длинных паузах диетологи советуют не отходить от обычного режима. Это поможет, как оказалось, держать прежний уровень массы мышц. Но и это нужно держать под контролем, так как процессы метаболизма при сохранении тренировок высокие. Больше потребляется, но и расходуется калорий.

    И если питание оставить прежним — тело не сможет расходовать излишек энергии, а скорее всего отложит его в ресурс. Также может возникнуть стресс при длительном отдыхе, так как занятия вызывают выброс эндорфинов.

    Двухнедельный пропуск вызывает понижение привычной выработки гормона, и это приведет к стрессам. Кроме этого, организм начинает испытывать напряжение. При сидячей работе, например, оно будет скапливаться в мышцах плеч и ног.

    А во время занятий оно обычно проходит. Если в период занятий не освободиться от стресса, то и отдых, и сон будут неполными. А пропуск также накопит нервную энергию, которая помешает глубокому сну.

    Заново начинать тренироваться нужно не так быстро. Хорошо сказывается общая атмосфера людей в зале, которые интенсивно занимаются. Это скорее возвратит на путь тренировок.

    Что лучше: тренировка на все тело или на отдельную группу мышц? | by BEST fit

    Большинство специалистов сходятся во мнении, что нет какой-то оптимальной тренировочной программы, которая бы подошла всем. Если кто-то будет утверждать обратное, игнорируйте такое мнение. Нужно организовать свои занятия таким образом, чтобы вы получали достаточно отдыха и не перетренировывались.

    Преимущества и недостатки есть как у сплит-программы (тренировки для отдельных мышечных групп), так и у проработки всех мышц за одну тренировку (full-body). Все зависит от ваших целей.

    Преимущества тренировки всех мышечных групп

    Такие тренировки полезны для спортсменов, которые хотят стать сильнее и не особо гонятся за набором мышечной массы, а также тех, кто находится на этапе «сушки» тела. Когда вы тренируете сразу все мышцы, работая с тяжелыми весами, вы получаете больший стимул к росту.

    Более продуктивное времяпровождение в тренажерном зале

    Не каждый может выделить время на то, чтобы посещать зал три и более раз в неделю. Тренировки сразу на все мышечные группы уменьшат время, которое вы проводите в зале, но не забывайте про нагрузку на приоритетные мышечные группы. Такие тренировки объединяют нагрузку на верхнюю и нижнюю части тела, что увеличит отклик на тренировки.

    Гормональный отклик на силовые тренировки

    Full-body тренировки задействуют большее количество мышечных групп, что увеличивает выработку тестостерона, гормона роста и инсулиноподобного фактора роста. Это усилит рост мышц и сжигание жира.

    Полезная статья: «Как с помощью питания повысить уровень тестостерона»

    Усиленное похудение и сжигание жира

    Интенсивные тренировки на все мышцы заставят вас работать в зале до седьмого пота. Круговые тренировки, малый интервал отдыха и базовые упражнения включат в работу больше мышечных групп, увеличат частоту сердечных сокращений и сожгут больше калорий. Примерно по такому же принципу построены тренировки по кроссфиту.

    Преимущества программы тренировок для отдельных мышечных групп (сплит)

    Эта схема отлично подойдет, если вы хотите накачать те или иные мышечные группы, увеличить максимальные показатели в базовых движениях или составить долгосрочную программу тренировок. Такие тренировки менее утомительны и идеально подходят для набора мышечной массы.

    Увеличение силовых показателей от силовых тренировок

    Тренировки на отдельные мышечные группы (грудь, спину, ноги и т. д.) занимают больше времени, но помогут вам добиться поставленных целей. К примеру, если вы хотите делать становую тягу больше 200 кг или жать лежа 140 кг, то тренироваться нужно соответствующе.

    Если вы весите 70 кг, а ваш разовый максимум в жиме лежа не превышает 100 кг, то full-body тренировки по 3 раза в неделю вам точно не подойдут. Вам подойдет тяжелая базовая работа над целевыми мышечными группами.

    Только будьте осторожны и не нагружайте одни мышцы сильнее других, это приведет к дисбалансу.

    Набор мышечной массы

    В период набора мышечной массы (ссылка на статью «Набор мышечной массы для новичков: питание, тренировки, добавки») мало толка от выполнения тренировок на все группы мышц. Не то чтобы это не сработает, но лучше сфокусироваться на отдельных мышечных группах. Также допустим вариант тренировки верха и низа в разные дни. Плюс сплит-программы также в том, что вы можете чередовать нагрузку, не нужно каждый день тренироваться до упада. Это важно для прогресса. Это и есть основа периодизации.

    Полезная статья: О пользе протеина для набора мышечной массы и похудения

    Меньше усталости от силовых тренировок

    Если ваша программа тренировок построена правильно, сплит-тренировки будут меньше вас утомлять, так как вы фокусируетесь только на одной или максимум двух мышечных группах.

    Тренировки на все тело расходуют больше калорий, постепенно это изнашивает организм, что навредит вашему тренировочному процессу.

    Это не значит, что вы будете полны сил после интенсивной тренировки ног. Но вы существенно снизите риск перетренировать свой организм, так как у ваших ног будет время для восстановления.

    Тренировать мышцы трапеции вместе с плечами или спиной?

    В этой статье мы выясним, какая тренировка мышц трапеции эффективнее для роста мышц: вместе с плечами или со спиной.

    Тренировать мышцы трапеции вместе с плечами или спиной?

    Тема этой статьи актуальна и постоянно обсуждаема профессиональными атлетами и тренерами. Ее касаются в любом тренажерном зале.

    Когда дело доходит до тренировки мышц трапеции, то одни предпочитают тренировать их вместе с плечами, а другие — со спиной. Какой из способов эффективнее? Объективного ответа пока нет, но разобраться в проблеме все же стоит.

     

    Что такое трапециевидные мышцы?

    Трапециевидные мышцы — это поверхностные мышцы верхней части спины. Крепятся к позвоночнику, начинаются от затылочной кости, достигают нижних позвонков и уходят в бок, к лопаткам. Основная функция трапеций заключается в поддержке рук и обеспечении движения лопаток.

    Название трапециевидные мышцы получили за счет своей формы, которая сильно напоминает трапецию.

     

    Когда используются мышцы трапеции?

    При более внимательном рассмотрении трапеций их можно разбить на три подраздела: верхний, средний и нижний. Верхняя часть трапеции, идущая от области шеи в бок, к ключице, поднимает плечи.

    Средняя часть, или поперечная, трапеции задействована при сведении лопаток вместе. Спорим, что вы никогда не задумывались о том, что тренируете трапецию при сведении лопаток, лежа на скамье? Средняя часть трапеции расположена посередине спины и простирается вбок, от основания шеи до края лопаточной кости.

    Что же касается низа трапеции, то он располагается между 4 и 12 отростками позвоночника, между лопаток и под ними, до конца позвоночника. Становится виден, если опустить лопатки вниз, держа руки прямо.

    Большинство считает, что развивать нужно только верх трапеции, который находится вверху спины. Но тренеры и медики скажут, что трапециевидные мышцы должны быть развиты полностью и равномерно, в противном случае может развиться дисбаланс, негативно влияющий на осанку, что позже способно вызвать проблемы с плечевым поясом.

     

    Упражнения для тренировки трапеций

    Несколько упражнений направлены именно на развитие трапециевидной области. Самое популярное из них выполняется очень просто: поднятие и опускание плеч, или шраги плечами.

    Стоя, руки вытянуты вдоль туловища, в руках отягощение, поднимите плечи вверх. Напрягите мышцы и медленно опустите плечи, пока не почувствуете растяжение трапеции. Довольно просто, верно? Самые популярные виды шрагов:

    • шраги стоя со штангой перед собой,
    • шраги стоя со штангой за спиной,
    • шраги с гантелями,
    • шраги в машине Смита.
     

    Уникальные упражнения для трапециевидных мышц

    Эти упражнения не имеют отношения к тренировкам трапеции с плечами или спиной, но внесут разнообразие в ваши тренировки, так как направлены на развитие трапеций. Это «прогулка фермера», тяга к лицу, горизонтальные подтягивания.

    «Прогулка фермера»: возьмите отягощение, которое будет чувствительно для ваших плеч. Ноги чуть согнуты, лопатки сведены. Прохаживайтесь короткими шагами. Во время шагов вы почувствуете растяжение трапеций. Кстати, если хотите проработать трапеции во время кардио, то это упражнение отлично подходит.

    Тяга к лицу с веревкой: наденьте на горизонтальный тросовый тренажер наконечник с канатом. Станьте прямо перед тренажером, возьмитесь руками за края каната. Отойдите от тренажера, держа канат, на расстояние вытянутых рук. Подтягивайте канат к подбородку, держа локти как можно выше. Сожмите трапеции и задние дельты, прежде чем отпустить канат. Это упражнение также отлично проработает ваши задние дельты.

    Горизонтальные подтягивания: установите бодибар или планку на высоте на уровне пояса. Возьмитесь широким хватом и займите положение под бодибаром так, словно вы держитесь за турник. Ноги держите прямо и опирайтесь на пятки. Подтянитесь как можно выше, до касания грудью бодибара, сводя лопатки. Опуститесь в исходное положение и повторите. Это тренирует широчайшие мышцы спины, ромбовидные и задние дельты.

     

    Тренировка трапеций со спиной

    Аргументом для тренировки трапеций вместе с мышцами спины является тот факт, что трапециевидные мышцы сами являются крупными мышцами спины, участвующими в поддержке позвоночника.

    Трапеции участвуют в основных упражнениях на спину, таких как мертвая тяга штанги на шестигранном грифе, тяга в раме, тяга штанги в наклоне.

    Так как трапеции работают во время выполнения этих базовых упражнений, есть смысл тренировать их вместе со спиной.

     

    Образец тренировки спины

    Упражнения

    Подходы

    Повторения

    1. Становая тяга

    5

    5

    2. Тяга штанги в наклоне

    3

    10–12

    3. Тяга гантели одной рукой к поясу в упоре

    3

    10–12

    4. Тяга блока вниз узким хватом

    3

    10–12

    5. Тяга нижнего блока прямыми руками

    3

    10–12

    6. Шраги со штангой

    3

    15

     

    Тренировка трапеций с плечами

    Аргументов для тренировки трапеций с плечами целых два. Профессиональные тяжелоатлеты и тренеры скажут вам, что трапеции настолько связаны с лопатками и задействованы в поддержке их движения, что тренировать их нужно вместе с дельтовидными мышцами.

    К тому же видимая часть трапеций находится рядом с плечами. Ничто не вдохновляет так, как пара широких округлых плеч и рельефные трапеции, опирающиеся на них сверху.

    Популярные упражнения для плечевого пояса, которые нагружают мышцы трапеции: тяга штанги к подбородку широким хватом, взятие штанги на грудь из виса, рывки и любые другие движения, которые требуют поднятия рук выше плеч. Этот пример тренировки показывает, как можно тренировать плечи с акцентом на трапеции.

     

    Образец тренировки плеч

    Упражнения

    Подходы

    Повторения

    1. Толчок штанги

    3

    6, 4, 2

    2. Подъем штанги перед собой стоя

    3

    8–10

    3. Подъем гантелей через стороны сидя

    3

    10

    4. Подъем гантелей через стороны на наклонной скамье

    3

    10

    5. Тяга штанги к подбородку широким хватом

    3

    15

    6. Шраги с гантелями

    3

    15

     

    А теперь ваше мнение

    Мы хотели бы услышать ваше мнение по следующим 5 вопросам.

    1. Что вы думаете о тренировке трапеции со спиной и плечами?
    2. Какой способ тренировки вы предпочитаете?
    3. Почему?
    4. Какие упражнения на трапецию ваши любимые?
    5. Есть ли что-нибудь, что вы хотите добавить?

    Мы будем рады вашим комментариям, в том числе и содержащим конструктивную критику, и со своей стороны постараемся поддержать разговор, отвечая на них.

    a:43:{s:16:»ADD_REVIEW_PLACE»;s:1:»1″;s:17:»BUTTON_BACKGROUND»;s:7:»#dbbfb9″;s:10:»CACHE_TIME»;s:8:»36000000″;s:10:»CACHE_TYPE»;s:1:»A»;s:26:»COMMENTS_TEXTBOX_MAXLENGTH»;s:4:»1000″;s:20:»COMPOSITE_FRAME_MODE»;s:1:»A»;s:20:»COMPOSITE_FRAME_TYPE»;s:4:»AUTO»;s:11:»DATE_FORMAT»;s:5:»d.m.Y»;s:21:»DEFAULT_RATING_ACTIVE»;s:1:»3″;s:12:»FIRST_ACTIVE»;s:1:»2″;s:10:»ID_ELEMENT»;s:4:»5206″;s:11:»INIT_JQUERY»;s:1:»N»;s:10:»MAX_RATING»;s:1:»5″;s:12:»NOTICE_EMAIL»;s:0:»»;s:13:»PRIMARY_COLOR»;s:7:»#a76e6e»;s:27:»QUESTIONS_TEXTBOX_MAXLENGTH»;s:4:»1000″;s:25:»REVIEWS_TEXTBOX_MAXLENGTH»;s:4:»1000″;s:13:»SHOW_COMMENTS»;s:1:»Y»;s:14:»SHOW_QUESTIONS»;s:1:»N»;s:12:»SHOW_REVIEWS»;s:1:»N»;s:18:»COMPONENT_TEMPLATE»;s:4:»blog»;s:17:»~ADD_REVIEW_PLACE»;s:1:»1″;s:18:»~BUTTON_BACKGROUND»;s:7:»#dbbfb9″;s:11:»~CACHE_TIME»;s:8:»36000000″;s:11:»~CACHE_TYPE»;s:1:»A»;s:27:»~COMMENTS_TEXTBOX_MAXLENGTH»;s:4:»1000″;s:21:»~COMPOSITE_FRAME_MODE»;s:1:»A»;s:21:»~COMPOSITE_FRAME_TYPE»;s:4:»AUTO»;s:12:»~DATE_FORMAT»;s:5:»d.m.Y»;s:22:»~DEFAULT_RATING_ACTIVE»;s:1:»3″;s:13:»~FIRST_ACTIVE»;s:1:»2″;s:11:»~ID_ELEMENT»;s:4:»5206″;s:12:»~INIT_JQUERY»;s:1:»N»;s:11:»~MAX_RATING»;s:1:»5″;s:13:»~NOTICE_EMAIL»;s:0:»»;s:14:»~PRIMARY_COLOR»;s:7:»#a76e6e»;s:28:»~QUESTIONS_TEXTBOX_MAXLENGTH»;s:4:»1000″;s:26:»~REVIEWS_TEXTBOX_MAXLENGTH»;s:4:»1000″;s:14:»~SHOW_COMMENTS»;s:1:»Y»;s:15:»~SHOW_QUESTIONS»;s:1:»N»;s:13:»~SHOW_REVIEWS»;s:1:»N»;s:19:»~COMPONENT_TEMPLATE»;s:4:»blog»;s:8:»TEMPLATE»;s:4:»blog»;}

    a:43:{s:16:»ADD_REVIEW_PLACE»;s:1:»1″;s:17:»BUTTON_BACKGROUND»;s:7:»#dbbfb9″;s:10:»CACHE_TIME»;s:8:»36000000″;s:10:»CACHE_TYPE»;s:1:»A»;s:26:»COMMENTS_TEXTBOX_MAXLENGTH»;s:4:»1000″;s:20:»COMPOSITE_FRAME_MODE»;s:1:»A»;s:20:»COMPOSITE_FRAME_TYPE»;s:4:»AUTO»;s:11:»DATE_FORMAT»;s:5:»d.m.Y»;s:21:»DEFAULT_RATING_ACTIVE»;s:1:»3″;s:12:»FIRST_ACTIVE»;s:1:»2″;s:10:»ID_ELEMENT»;s:4:»5206″;s:11:»INIT_JQUERY»;s:1:»N»;s:10:»MAX_RATING»;s:1:»5″;s:12:»NOTICE_EMAIL»;s:0:»»;s:13:»PRIMARY_COLOR»;s:7:»#a76e6e»;s:27:»QUESTIONS_TEXTBOX_MAXLENGTH»;s:4:»1000″;s:25:»REVIEWS_TEXTBOX_MAXLENGTH»;s:4:»1000″;s:13:»SHOW_COMMENTS»;s:1:»Y»;s:14:»SHOW_QUESTIONS»;s:1:»N»;s:12:»SHOW_REVIEWS»;s:1:»N»;s:18:»COMPONENT_TEMPLATE»;s:4:»blog»;s:17:»~ADD_REVIEW_PLACE»;s:1:»1″;s:18:»~BUTTON_BACKGROUND»;s:7:»#dbbfb9″;s:11:»~CACHE_TIME»;s:8:»36000000″;s:11:»~CACHE_TYPE»;s:1:»A»;s:27:»~COMMENTS_TEXTBOX_MAXLENGTH»;s:4:»1000″;s:21:»~COMPOSITE_FRAME_MODE»;s:1:»A»;s:21:»~COMPOSITE_FRAME_TYPE»;s:4:»AUTO»;s:12:»~DATE_FORMAT»;s:5:»d.m.Y»;s:22:»~DEFAULT_RATING_ACTIVE»;s:1:»3″;s:13:»~FIRST_ACTIVE»;s:1:»2″;s:11:»~ID_ELEMENT»;s:4:»5206″;s:12:»~INIT_JQUERY»;s:1:»N»;s:11:»~MAX_RATING»;s:1:»5″;s:13:»~NOTICE_EMAIL»;s:0:»»;s:14:»~PRIMARY_COLOR»;s:7:»#a76e6e»;s:28:»~QUESTIONS_TEXTBOX_MAXLENGTH»;s:4:»1000″;s:26:»~REVIEWS_TEXTBOX_MAXLENGTH»;s:4:»1000″;s:14:»~SHOW_COMMENTS»;s:1:»Y»;s:15:»~SHOW_QUESTIONS»;s:1:»N»;s:13:»~SHOW_REVIEWS»;s:1:»N»;s:19:»~COMPONENT_TEMPLATE»;s:4:»blog»;s:8:»TEMPLATE»;s:4:»blog»;}

    5 мышц, которые можно тренировать каждый день

    Тренируйте эти группы мышц каждый день

    Одна из первых вещей, которую люди узнают, посещая тренажерный зал, — это то, что им не следует тренировать одну и ту же группу мышц каждый день. Хотя это утверждение в основном верно, есть некоторые исключения.

    Вы не тренируете одну и ту же группу мышц каждый день, потому что вашим мышцам нужно время, чтобы восстановиться и стать больше после тренировок.

    В то время как большим группам мышц, таким как спина, грудь и ноги, может потребоваться относительно больше времени, меньшие группы мышц могут быть готовы к действию в течение 24 часов после тренировки.

    Телята

    Икры — одна из самых устойчивых мышц и может быть самым слабым звеном для телосложения многих людей. Икры представляют собой небольшую группу мышц и быстро восстанавливаются, потому что они привыкли терпеть много ударов.

    Основное правило тренировки телят: если они не болят, дрессируйте их. Когда вы постоянно занимаетесь стоянием, ходьбой и бегом, ваши икры могут сделать гораздо больше, чем несколько подходов по 10-15 повторений.

    Предплечья

    Предплечья — одна из самых недооцененных мышц, и лишь некоторые люди включают их в свои программы тренировок. Симметричные предплечья, бицепсы и трицепсы могут сделать ваши руки больше и сильнее.

    Если у вас тонкие предплечья, вам следует подумать о выполнении сгибаний запястий сверху и снизу в конце тренировки. Выполнения трех подходов по 15-20 повторений должно хватить, чтобы вытянуть предплечья «Попай».

    Шея

    Давайте будем честными, как часто вы тренируете шею? Не стесняйтесь своего ответа, потому что у большинства людей на этот вопрос будет один и тот же ответ — не много.Мускулистая верхняя часть тела с шеей-зубочисткой — одно из самых неприятных зрелищ.

    Самое лучшее в тренировке шеи — это то, что вы можете добиться отличных результатов без особых усилий. Вам не нужно делать типичные три подхода по пять упражнений для тренировки шеи. Выполнение пары упражнений в конце тренировки может превратить зубочистку в деревянное бревно.

    Abs

    Abs, вероятно, самая очевидная запись в этом списке. Скорее всего, вы уже тренируете пресс каждый день.Если вы тренируете мышцы брюшного пресса каждый день, вы должны нацеливать их, разделив их на такие части, как верхний, нижний, средний пресс и косые мышцы живота.

    Тренировка разных разделов каждый день даст другим разделам время восстановиться после тренировок. Выполнение упражнений на укрепление кора, таких как планка, может помочь в наращивании общей силы для сложных упражнений.

    Работа на ленте

    Выполнение упражнений с лентами — отличный способ улучшить подвижность, разогреться перед тренировкой и избавиться от молочной кислоты после ее окончания.Бинты работают великолепно, так как они практически не повреждают мышцы и могут ускорить процесс восстановления.

    Упражнения с бинтами также могут помочь улучшить связь между мозгом и мышцами, улучшить накачку и устранить слабые места. Растягивание с лентой, жим на трицепс с лентой, сгибание на бицепс с лентой, сгибание ног с лентой и вытягивание с лентой — вот несколько примеров упражнений, которые можно выполнять с эластичными лентами.

    Изображение заголовка любезно предоставлено Envato Elements.


    Тренируете ли вы какую-либо группу мышц каждый день? Дайте нам знать в комментариях ниже.Также не забудьте подписаться на Generation Iron в Facebook, Twitter и Instagram.

    Какие мышцы мне следует работать вместе?

    Как тренеры, мы часто слышим, что самая большая жалоба при начале программы тренировок — «Я не знаю, с чего начать». Или «Как мне узнать, какие группы мышц тренировать вместе?» Хорошие новости: действительно нет неправильного способа тренировать группы мышц (конечно, с хорошей техникой), но мы можем помочь вам выбрать программу, которая подходит именно вам, и объяснить, почему определенные группы мышц часто работают вместе. .

    Вот некоторые комбинации мышц, которые часто тренируются вместе:

    1. Грудь, плечи и трицепсы
    2. Спина и бицепсы
    3. Подколенные сухожилия, икры и ягодицы

    Грудь и трицепс — отличная комбинация, потому что обе эти мышцы являются «толкающими» мышцами, то есть они активируются, когда вам нужно оттолкнуть что-то от своего тела. Например, во время жима от груди ваш трицепс является второстепенным двигателем в этом упражнении. Работа над трицепсами при работе над грудью дает вашим трицепсам двойную тренировку.Три основных способа задействовать эти группы мышц — это жим от груди, жим от плеч и разгибание трицепса.
    Спина и бицепс — еще одна отличная комбинация для совместной работы. Спина и бицепс — это «тянущие» мышцы, то есть они активны, когда вам нужно подтянуть что-то ближе к своему телу. Если вы тренируете бицепсы в день, когда тренируете спину, ваши бицепсы тренируются даже лучше, чем если бы вы в этот день пропустили бицепсы. Вы можете задействовать эти мышцы, выполняя сгибания рук на бицепс и тяги гантелей.
    Если вы хотите узнать больше о разнице между толкающими и тянущими мышцами, посетите блог на указанном ниже веб-сайте:

    https://www.pumpfitclub.com/exercise/differences-push-pull-muscles/

    Еще одна замечательная программа задействует все мышцы ног. Подколенные сухожилия и ягодицы — это мышцы задней части ног. Они часто работают вместе, чтобы двигать вашим телом вверх и вниз. Некоторые упражнения изолируют каждую мышцу, но некоторые упражнения задействуют все группы мышц одновременно одним большим сложным движением.Вы можете проработать эти мышцы вместе, выполняя такие упражнения, как перекатывание подколенного сухожилия, приседания и ягодичные мосты со штангой.

    Вопреки распространенному мнению, ваше «ядро» — это НЕ только мышцы пресса! «Многие мышцы скрыты под внешней мускулатурой, которую обычно тренируют люди. К более глубоким мышцам относятся поперечные мышцы живота, мультифидус, диафрагма, тазовое дно и многие другие более глубокие мышцы ». — Breakingmuscle.com. Сильный стержень важен для устойчивости и баланса, а также для снижения риска травм.Вместо того, чтобы делать бесконечные скручивания, попробуйте выполнять функциональные упражнения, такие как планка, русские скручивания и растягивание пальцев ног. Сделайте тренировку более сложной, добавив веса к своему распорядку или сделав больше подходов и повторений.
    Помните, что каждое упражнение, которое вы делаете, вы должны держать задействованными основные мышцы. Даже когда вы лежите и делаете жим от груди, мышцы пресса должны быть напряжены! Это гарантирует, что вы получаете отличную тренировку пресса, даже если не уделяете им внимания, а также обеспечивает стабильную основу, особенно при использовании большего веса.Если вы занимаетесь более легкими весами и нацелены на мышечную выносливость, подойдет и все тело. Но если вы занимаетесь тяжелой атлетикой, не забудьте дать себе достаточно времени для восстановления между подходами и даже между днями.

    Не рекомендуется поднимать одни и те же группы мышц два дня подряд, если вы поднимаете тяжести. Отдых так же важен, как и упражнения, потому что рост мышц на самом деле происходит во время периода отдыха, когда у сломанных мышечных волокон есть шанс восстановиться.Дайте себе хотя бы один день отдыха в неделю или дни активного восстановления между силовыми тренировками. Активное восстановление означает тренировку низкой интенсивности на следующий день после тяжелой или сложной тренировки, такой как бег трусцой или плавание.
    И не забывайте добавлять кардио к тренировкам. Только убедитесь, что не используете кардио в качестве единственного источника упражнений. Есть так много причин, по которым поднятие тяжестей более полезно для тренировки, чем только кардио. Вы всегда можете прочитать все об этих причинах в нашем другом блоге под названием «Почему кардио не всегда является ответом на потерю жира.
    Просто помните, что это рекомендации, и не существует «универсальной модели». Просто тренировки полезны для тела, поэтому не позволяйте себе одолевать мысли «тренирую ли я правильные мышцы вместе?». Делайте то, что считаете лучшим для себя. Если вы когда-нибудь не уверены, достаточно ли вашей программы тренировок, свяжитесь с одним из наших замечательных тренеров и запланируйте бесплатное занятие прямо сегодня!

    Следует ли тренировать большие мышцы иначе, чем маленькие?

    Многие люди не определяют тренировочный объем индивидуально для каждой группы мышц.Некоторые люди приближаются к сокращенному способу выполнения упражнений на толкание и тягу, но эта классификация не подходит для вращательных упражнений, таких как упражнения на грудную клетку, которые не являются ни тяговым движением, ни мухи, и таких упражнений, как приседания со штангой, которые технически являются тягой, но тренируют мышцы. обычно ассоциируется с прессингом (например, квадрицепсы). Они также не принимают во внимание важные биомеханические различия между внешне похожими жимами или тягами, например, жим гантелей не активирует трицепсы в такой степени, как жим штанги из-за более открытой кинетической цепи.В результате того, что объем тренировки не оптимизирован для каждой мышцы в отдельности, многие распространенные программы тренировок, такие как «Стартовая сила» или 5 × 5, крайне несбалансированы с точки зрения объема на группу мышц.

    Благодаря новому исследованию объема тренировок, проведенному за последний год, люди стали уделять больше внимания тому, как считать объем тренировок. Возникает вопрос: при определении тренировочного объема для каждой мышцы, должен ли он быть одинаковым для всех мышц, или одни мышцы требуют большего объема, чем другие, для максимального роста?

    Обучение малое vs.большие мышцы: теория

    Есть несколько причин, по которым вы хотите придать разным группам мышц больший объем, чем другим, в частности, насколько важны эти мышцы для вас и их тренировочного прогресса. В этой статье я остановлюсь на одном конкретном факторе: размере мышц. Существует множество теорий о том, как размер мышц влияет на объем и частоту тренировок, но на данный момент они в основном спекулятивны:

    • Тренировка больших групп мышц теоретически может вызвать большее утомление центральной нервной системы, что снижает оптимальную частоту тренировок, но, как я ранее показал, утомляемость ЦНС переоценивается.

    • Тренировка больших групп мышц может потреблять больше ресурсов восстановления, поэтому требуется меньший объем или частота тренировок. Однако я не вижу, какие центральные ресурсы здесь будут иметь значение. Метаболические факторы, такие как кислород и кровоток, должны восстановиться в течение нескольких минут, поэтому они не должны влиять на частоту тренировок.

    • Маленькие группы мышц должны иметь меньше мышечных волокон с меньшей функциональной компартментализацией, поэтому им может потребоваться меньший объем для стимуляции всех мышечных волокон.Для меня это имеет смысл, но количество мышечных функций и их размер не коррелируют в полной мере. Например, квадрицепсы огромны, но относительно просты по структуре, тогда как подколенные сухожилия меньше (данные ниже), но гораздо более разделены на части.

    Чего я нигде не видел, так это всестороннего анализа данных, чтобы увидеть, какие теории верны. Итак, давайте посмотрим на факты, но прежде чем мы это сделаем, мы должны сначала классифицировать, какие мышцы маленькие, а какие большие, потому что представление большинства людей об этом основано исключительно на внешнем виде мышц.Оценка размера мышцы на основании того, насколько она велика снаружи, очень необъективна, потому что вы не можете увидеть, как далеко она уходит внутрь. Это похоже на попытку угадать, сколько квадратных футов в доме, глядя только на главный вход.

    Например, многие люди думают, что широчайшие — это большая группа мышц, потому что они покрывают большую площадь поверхности, но их общий объем мышц относительно невелик, поскольку они представляют собой относительно тонкий слой мышц. С другой стороны, икры обычно рассматриваются как небольшая группа мышц, но на самом деле они очень большие, поскольку покрывают большую часть внутренней части нашей голени, а не только ту часть, которую вы видите на поверхности.

    Какие мышцы большие, а какие маленькие?

    Моя исследовательская группа собрала данные всех исследований, которые мы смогли найти, которые измеряли или оценивали объем мышц в различных мышцах человеческого тела. Эти данные относятся к нетренированным лицам, чтобы исключить искажение показателей на основании опыта обучения. Очевидно, что в противном случае более тренированные мышцы имели бы тенденцию быть больше, чем менее тренированные. Многие исследования проводятся на трупах, поскольку живые люди часто не хотят подвергаться вскрытию.Наиболее актуальными исследованиями являются Holzbauer et al. (2007), Lube et al. (2016), Гарнер и Пэнди (2003), Вуд и др. (1989), Veeger et al. (1991), Bassett et al. (1990), Winters & Stark (1988), An et al. (1981), Veegers et al. (1997) и Cutts et al. (1991).

    Большинство измерений хорошо согласуются друг с другом. На следующих изображениях мы показываем средневзвешенный объем мышц каждой основной группы мышц для мужчин и женщин. В некоторых исследованиях сообщалось только о смешанных гендерных измерениях.Мы сгруппировали их с данными по мужчинам, потому что — что, возможно, удивительно — соотношение размеров различных мышц у мужчин и женщин примерно одинаково. Причина, по которой у женщин кажется более толстой нижней части тела и большей попой, по-видимому, в основном потому, что они накапливают там больше жира, а не потому, что эти мышцы более развиты.

    Данные рисуют совершенно иную картину, чем многие интуитивно предполагают:

    1. Квадрицепсы — самая большая группа мышц в организме, как у мужчин, так и у женщин.Они примерно в два раза больше, чем занявшие второе место.

    2. Следующая по величине группа мышц — это связка между большой ягодичной мышцей и икроножными мышцами. Да, икры огромные. Тем не менее, это может быть отчасти потому, что икры у нетренированных людей уже относительно более развиты, чем другие мышцы, поскольку они используются всякий раз, когда вы стоите на ногах, а камбаловидная мышца является чрезвычайно медленной и доминирующей, поэтому она хорошо реагирует тренировкам на выносливость. Так что у тренированных людей икры, вероятно, несколько падают.К тому же, если включить среднюю и малую ягодичные мышцы, ягодичные мышцы намного больше, поэтому мы можем сказать, что ягодицы на втором месте, а икры на третьем.

    3. Затем закройте после того, как у нас есть подколенные сухожилия, прежде чем мы перейдем к верхней части тела, которая имеет гораздо меньшие группы мышц, чем нижняя часть тела.

    4. Дельты — самая большая группа мышц верхней части тела. Бицепс самый маленький. Между ними большинство мышц примерно одинакового размера, включая трапеции, грудные мышцы, трицепсы и широчайшие.

    Теперь, когда у вас есть классификация малых и больших мышц, давайте посмотрим, что говорят данные о том, как мы должны тренировать эти мышцы.

    Тренировка больших и малых мышц: объем

    Один из способов выяснить, нужно ли тренировать большие и маленькие мышцы по-разному, — это посмотреть, как они реагируют на изменение тренировочного объема. Одни мышцы получают больше пользы от больших объемов, чем другие? У нас есть 13 исследований, в которых измерялось влияние различных объемов тренировок на разные мышцы при сохранении постоянной частоты тренировок и других переменных программы: Barbalho et al.(2018), Schoenfeld et al. (2018), Островски и др. (1997), Radaelli et al. (2015), Amirthalingam et al. (2017), Роннестад и др. (2007), Боттаро и др. (2011), Radaelli et al. (2013), Radaelli et al. (2014), Rhea et al. (2002), Barbalho et al. (2019), Hackett et al. (2018) и McBride et al. (2003).

    На сегодняшний день наиболее изученными мышцами являются бицепсы, трицепсы и квадрицепсы, потому что размер мышц конечностей намного легче измерить, чем размер мышц туловища, но у нас также есть данные о груди, ягодицах и подколенных сухожилиях и 2 исследования, которые позволяют нам это сделать. сравните все руки vs.целиком ноги.

    В 11 из этих 13 исследований все мышцы одинаково реагировали на изменение тренировочного объема: не было значительной разницы во влиянии тренировочного объема на рост мышц. Независимо от того, значительно увеличился тренировочный объем, уменьшился или не повлиял на рост мышц, этот эффект не отличался в зависимости от размера различных измеренных мышц. Это было верно для 9 из 13 исследований, даже если брать только исходные значения процентного роста мышц, а не просто учитывать статистическую значимость: все максимальные значения роста мышц были в одной и той же группе объема в этих исследованиях.Подобная реакция на тренировочный объем убедительно свидетельствует о том, что большие и маленькие мышцы следует тренировать с одинаковым объемом.

    Однако, если мы посмотрим на 4 исследования, которые не обнаружили полностью однородного влияния тренировочного объема на рост мышц между разными мышцами, 3 из них подтверждают определенную тенденцию.

    1. Наверное, не стоит смотреть на первое исследование. В Amirthalingam et al. (2017) в исследовании German Volume Training, трицепсы и подколенные сухожилия росли почти в два раза быстрее при увеличении объема, тогда как квадрицепсы и бицепсы росли более чем в два раза быстрее при более низком объеме тренировки.Объем тренировки не был равномерно сбалансирован между мышцами в этом исследовании, и объем тренировки был изменен только для основных движений. Поскольку тренировочный объем обычно имеет оптимальную U-образную связь с тренировочным объемом — чем больше, тем лучше до определенного момента, после которого он начинает становиться вредным, — это исследование не позволяет нам справедливо сравнить влияние тренировочного объема на эти мышцы.
    Как бы то ни было, бицепсы и квадрицепсы имели одинаковый оптимальный объем, и это были самые маленькие и самые большие мышцы в теле, поэтому эти данные не подтверждают никакого влияния размера мышцы на оптимальный тренировочный объем.

    2. Наиболее интересным исключением является Radaelli et al. (2014). В этом исследовании квадрицепсы росли быстрее в группе с большим объемом (17% против 13%), тогда как бицепсы — нет: на самом деле они росли незначительно медленнее в группе с большим объемом: 15% против 16%. «Большой объем» составлял 3 подхода на упражнение два раза в неделю, и участниками были нетренированные пожилые женщины, поэтому, возможно, квадрицепсы лучше реагировали на больший объем, поскольку они были менее тренированы. Это исследование не является самым убедительным доказательством, но оно проходило под надлежащим контролем и длилось 20 недель, так что это открытие подтверждает, что оптимальный тренировочный объем для бицепсов ниже, чем для квадрицепсов.

    3. В Bottaro et al. (2011), тренировочный объем не оказал существенного влияния на рост мышц у нетренированных мужчин. Хотя это и не является статистически значимым, группа квадрицепсов с меньшим объемом потеряла 2,9% мышц (они делали только 2 подхода в неделю, но это все еще впечатляющий подвиг — потерять мышцы для нетренированного человека…), тогда как группа квадрицепсов с большим объемом набрала 2,5% мышц. Бицепсы показали противоположную картину: 7,2% роста в группе из 1 подхода и 5,9% в группе из 3 подходов. Таким образом, это исследование слабо поддерживает Radaelli et al.(2014), что у квадрицепсов может быть более высокий оптимальный тренировочный объем, чем у бицепсов.

    4. В McBride et al. (2003), снова неподготовленным мужчинам и женщинам каким-то образом удалось добиться нулевого результата за 12-недельное исследование. Вот почему вы должны скептически относиться к сторонникам малого объема, цитирующим подобные исследования: эти субъекты, очевидно, бездельничали. Безжировая масса рук и ног существенно не увеличилась независимо от того, были ли участники в группе из 1 или 6 подходов. И когда я говорю о нулевой прибыли, я имею в виду это.Скорость роста мышц составляла 0% для групп с меньшим объемом в обеих мышцах. Однако единственный рост мышц произошел в группе из 6 подходов в квадрицепсах: 5,1%. Итак, снова у нас есть некоторые доказательства в пользу более высокого оптимального объема квадрицепсов, даже у людей с меньшим позвоночником, чем у медузы.

    Достаточно ли этих трех исследований, чтобы сформировать тренд? Ниже мы построили средние темпы роста мышц по всем исследованиям в 3 наиболее изученных мышцах — квадрицепсы, трицепсы и бицепсы — для групп с наименьшим и наибольшим объемом каждого исследования.Похоже, что тенденция есть. В среднем квадрицепсы и трицепсы набирают больше мышц, когда вы тренируете их с большим объемом, однако скорость роста бицепсов в среднем снижалась с увеличением тренировочных объемов.

    Плохая реакция бицепса на большие объемы вряд ли будет результатом его размера, потому что трицепс почти одинаково реагирует на квадрицепсы, несмотря на огромную разницу в размерах. Более того, все доступные данные отдельных исследований груди, ягодиц и подколенных сухожилий подтверждают, что эти мышцы одинаково реагируют на разные объемы тренировок, даже если они сильно различаются по размеру (грудные мышцы крошечные по сравнению с мышцами нижней части тела).У нас также есть обширная литература о влиянии добавления работы на изоляцию рук к комплексным упражнениям. В этой литературе обычно не отмечается небольшого улучшения мышечного роста от добавления работы по изоляции рук, особенно для бицепса. Примечательно, что Gentil et al. (2013) обнаружили, что добавление 6 подходов в неделю изолирующей работы на бицепс к 6 подходам тяги вниз не привело к значительному увеличению толщины мышц сгибателей локтя. Общие литературные средние величины эффекта подтверждают преимущество работы с изоляцией рук перед сложной, но она меньше и более непоследовательна, чем вы обычно слышите.Частично это может быть связано с плохой реакцией бицепса на большие объемы.

    В целом, 10 из 13 исследований подтверждают, что мы должны тренировать все мышцы с одинаковым объемом. 1 исследование показало, что бицепс имеет более низкий оптимальный объем, чем квадрицепсы, и еще 2 исследования не имеют существенной тенденции в этом же направлении. При усреднении результатов всех исследований оказывается, что бицепсы хуже реагируют на более высокие объемы тренировок, чем квадрицепсы и трицепсы. Исследования по добавлению изолирующей работы на бицепс к комплексным упражнениям также обнаруживают лишь слабые преимущества.Грудь, подколенные сухожилия и ягодицы реагируют на тренировочный объем так же, как трицепсы и квадрицепсы, поэтому бицепсы, по всей видимости, являются исключением, а не мелкими мышцами в целом.

    Тренировка больших и малых мышц: частота

    Мы можем проводить тот же анализ с частотой тренировок. Некоторые люди утверждают, что меньшие мышцы восстанавливаются быстрее из-за того, что им требуется меньше ресурсов для восстановления, хотя мне неясно, какие именно ресурсы это будут, поскольку такие факторы, как кровоток, не должны быть ограничивающим фактором восстановления между тренировками.

    У нас есть 5 исследований, в которых измерялось влияние различных частот тренировок на рост мышц при сохранении постоянных всех других переменных программы, включая набор и общий объем работы: Saric et al. (2018), Schoenfeld et al. (2015), Юэ и др. (2018), Арази и Асади и др. (2011) и Lasevicius et al. (2019).

    Во всех этих исследованиях бицепсы, трицепсы и квадрицепсы одинаково реагировали на разные частоты, и в 4 из 5 исследований максимальный рост мышц произошел в одной и той же группе частот, даже без учета статистической значимости.Saric et al. (2018) — единственное почти исключение: квадрицепсы не значительно лучше реагировали на более высокую частоту тренировок, чем бицепсы и трицепсы. Более того, бицепс набирал значительную мышечную массу только с меньшей частотой. Таким образом, это исследование дает слабые доказательства того, что квадрицепсы лучше реагируют на более частые тренировки, чем бицепсы.

    Чтобы посмотреть, есть ли какая-то общая тенденция, подтверждающая, что мы должны тренировать квадрицепсы чаще, чем бицепсы, мы построили график среднего роста мышц для 3 мышц в нижнем и нижнем отделах.группы высокой частоты ниже. Все линии проходят примерно в одном направлении, что указывает на то, что влияние частоты тренировок одинаково для всех мышц. Тем не менее, кажется, что квадрицепсы немного лучше, чем трицепсы и бицепсы, реагируют на частые тренировки.

    Интересно, что общая тенденция нисходящая, предполагая, что более высокие частоты при том же общем тренировочном тоннаже могут быть вредными. Так стоит ли нам тренироваться с более низкими частотами?

    Вероятно, нет, потому что за пределами лаборатории вы не находитесь в условиях, приравненных к тоннажу.Поддержание такого же тоннажа обычно возможно только в том случае, если испытуемые тренируются на субмаксимальном уровне. Если вы распределите упражнения или подходы на большее количество тренировок, у вас должно получиться больше повторений. Я уже много раз говорил об этом на своем сайте и в обзорах исследований в социальных сетях, поэтому я еще раз кратко объясню это на примере: когда вы можете сделать больше повторений в жиме лежа на 80% от 1ПМ: когда вы делать 10 подходов в понедельник (национальный день жима лежа) или когда вы делаете 5 подходов в понедельник и 5 подходов в четверг?

    Если вы ответили 10 сетов в понедельник, вы вообще поднимаете?

    Распределение работы в течение недели снижает негативное влияние усталости на производительность, тем самым позволяя вам поднимать более тяжелые веса или делать больше повторений с тем же весом.Таким образом, более высокие частоты в практических условиях обычно увеличивают общий тренировочный тоннаж (подходы x повторений x вес). Давайте посмотрим на эффект от частоты тренировок, когда мы сохраняем количество подходов прежним, но увеличиваем тоннаж. У нас есть 5 исследований, которые соответствуют этим критериям и измеряют несколько мышц: Zaroni et al. (2018), Феррари и др. (2013), Искьердо и др. (2005), Gomes et al. (2018) и Brigatto et al. (2018).

    В отличие от исследований, приравненных к тоннажу, в этих более естественных условиях средние темпы роста мышц были самыми высокими для групп с более высокой частотой для всех мышц: бицепса (6.5% против 4,6%), трицепс (8,5% против 5,7%), а также квадрицепсы (10,5% против 8,1%), см. График ниже. Это поддерживает потенциальную выгоду от более высокой частоты тренировок, которая опосредована увеличением тренировочного объема. Я уже много раз обсуждал это на своем сайте и в социальных сетях, поэтому не буду вдаваться в подробности здесь.

    Вернемся к теме этой статьи: эти данные подтверждают, что размер мышц не влияет на реакцию на частоту тренировок. В этих данных большие и маленькие мышцы одинаково реагируют на разную частоту тренировок.

    Большинство из 5 отдельных исследований подтверждают общую тенденцию. Общая тенденция свидетельствует о том, что все 3 мышцы получают одинаковую пользу от более частых тренировок: больше работы и, следовательно, больше роста. 2 из 5 исследований полностью подтверждают это: эффект частоты тренировок был одинаковым для всех мышц, и все мышцы достигли максимального мышечного роста в одной и той же группе.

    У нас также есть исследование Stec et al. (2017), которые изучали влияние разных частот тренировок без сохранения установленного объема.Бедра и руки лучше реагировали на более частые объемы.

    В остальных 3 исследованиях Brigatto et al. (2018), Gomes et al. (2018) и Ferrari et al. (2013) не было статистически значимой разницы в росте мышц между низкой и высокой частотой тренировок. Brigatto et al. обнаружили, что бицепсы не достигли значительного роста в группе с низкой частотой, в отличие от трицепсов и квадрицепсов. Феррари и др. также обнаружил, что бицепсы достигли наибольшего роста в группе с низкими частотами, в отличие от квадрицепсов.Однако Gomes et al. обнаружили, что руки достигли лучших результатов в группе с высокой частотой, тогда как ноги достигли лучшего роста в группе с низкой частотой.

    Основываясь на результатах этих индивидуальных исследований, можно сказать, что здесь снова наблюдается очень слабая тенденция, подтверждающая, что бицепсы меньше выигрывают от более частых тренировок, чем трицепсы и квадрицепсы. Однако в среднем это полностью уравновешивает, и ни одно из исследований не обнаружило статистически значимой разницы в реакции этих мышц на частоту тренировок.

    Заключение

    В целом, данные показывают, что все мышцы одинаково реагируют на тренировочный объем и частоту тренировок. Чем больше тренировочный объем, тем лучше до точки, которая зависит от способности к восстановлению. Более высокие частоты тренировок могут быть полезны, но обычно только в том случае, если они приводят к более высокому общему недельному тренировочному тоннажу. Таким образом, мы, вероятно, можем тренировать все мышцы одинаково, учитывая, что они одинаково развиты, вы уделяете им одинаковое внимание и т. Д.Относительный размер различных мышц не влияет на то, как их следует тренировать. Фактически, большинство теорий, основанных на относительном размере мышц, с самого начала неправильно классифицируют мышцы на большие или маленькие. Например, дельты обычно называют небольшой группой мышц, но объективно они являются крупнейшей основной группой мышц верхней части тела. Точно так же икры обычно называют небольшой группой мышц, но на самом деле они являются одной из самых больших групп мышц во всем теле.

    Однако есть тенденция к тому, что бицепсы хуже реагируют на более высокие частоты тренировок и особенно тренировочные объемы, чем квадрицепсы.Это вряд ли связано с размером этих мышц, потому что трицепс реагирует аналогично квадрицепсу, но по размеру гораздо ближе к бицепсу, чем квадрицепс. Кроме того, все доступные данные свидетельствуют о том, что поддержку грудной клетки, ягодиц и подколенных сухожилий можно тренировать так же, как и другие мышцы, несмотря на то, что они имеют очень разные относительные размеры. Так что в бицепсах может быть что-то уникальное, из-за чего они не получают особой выгоды от большей тренировки. Пока у нас не будет дополнительных исследований и правдоподобного механизма, чтобы объяснить это, я не стал бы радикально менять вашу тренировку бицепса, но, по крайней мере, данные твердо подтверждают, что мы не должны тренировать бицепс на больше, чем на , чем другие мышцы.

    Если вам нравится всесторонний анализ данных об оптимальных тренировках и диете, вам понравится наша онлайн-сертификация PT. Он наполнен материалами, которых вы больше нигде не найдете. Регистрация открылась только 4 октября, и все в порядке очереди.

    Какие мышцы лучше всего тренировать в один день?

    Выбор мышц для совместной тренировки во многом зависит от вашего опыта, целей и количества дней в неделю, которые вы посвящаете силовым тренировкам.Существует несколько школ мысли о том, как тренировать мышцы и какие из них хорошо сочетаются друг с другом, но самые важные шаги, которые вы должны предпринять, — это отдыхать мышцы между тренировками и прислушиваться к своему телу, чтобы избежать перетренированности.

    Определите свои цели

    Прежде чем приступить к новому плану тренировок, лучше всего определить, каковы ваши цели и как часто вы можете тренироваться. Для новичков и тех, кто тренируется только с отягощениями два-три раза в неделю, тренировка всего тела — лучший вариант, чтобы все группы мышц тренировались как минимум дважды в неделю.Те, кто может тренироваться четыре-пять дней в неделю, обычно имеют более продвинутый опыт упражнений и стремятся нарастить мышцы или повысить тонус. Они лучше подходят для сплит-режима, например, чередования дней для верхней и нижней части тела, или даже для более изолированных мышечных дней, таких как грудь и бицепсы в один день, ноги и корпус — в другой.

    Программа для всего тела

    Программа для всего тела может принести пользу всем уровням физической подготовки. Если вы занимаетесь силовыми тренировками только два-три дня в неделю, тренировка для всего тела является идеальной, потому что она прорабатывает каждую группу мышц в небольших количествах, поэтому вы не рискуете перетренироваться и почувствовать боль перед следующей тренировкой.Сначала проработайте большие группы мышц, такие как ноги, грудь и спина. Например, вы можете сделать становую тягу с тягой в наклоне, чтобы проработать сразу несколько групп мышц. Выполняйте одно-два упражнения на каждую группу мышц. Затем сделайте это с меньшими группами мышц, такими как трицепсы, бицепсы, плечи и кора. Чем больше групп мышц вы задействуете, тем тяжелее ваше тело будет работать за более короткий промежуток времени. Старайтесь, чтобы две группы мышц работали вместе одновременно. Другие примеры включают выпады со сгибаниями на бицепс, жимы от груди с подъемом ног и приседания с жимами от плеч.

    Верхняя и нижняя части тела

    Еще один способ разделить ваш распорядок — это делать в один день всю верхнюю часть тела, а в следующий — нижнюю. Ваша верхняя часть тела включает грудь, спину, бицепсы, трицепсы, предплечья, плечи и пресс. Нижняя часть тела состоит из четырехглавой мышцы, подколенных сухожилий, ягодиц, поясницы и икры. Чтобы дать достаточно времени для отдыха, выполняйте упражнения на верхнюю часть тела, например, в понедельник, затем на нижнюю часть тела во вторник, отдыхайте в среду, затем повторите, начиная с четверга, чтобы каждая группа мышц получала две тренировки в неделю.Если вы новичок, придерживайтесь только одной тренировки на каждую группу мышц в неделю, чтобы избежать перетренированности. По мере того, как ваша сила и выносливость улучшатся, начните добавлять еще один или два дня.

    Сплит-программа

    Если вы более продвинуты и несколько дней посвящаете силовым тренировкам, лучше всего разделить мышцы на еще меньшие группы. Например, если вы тренируетесь пять дней в неделю, начните первый день с груди и бицепсов. Второй день может быть на спину и трицепс, третий день — на ноги и корпус, четвертый день — на плечи, а пятый день может быть на другой день для ног и кора, или на второй день для группы мышц, на которую вы более активно ориентируетесь для роста.Некоторые программы любят включать в себя аналогичные рабочие группы мышц, такие как грудная клетка и трицепс, а не бицепс, потому что упражнения на грудь также воздействуют на трицепс. Однако, чтобы максимально увеличить мышечную активность, вы должны работать с противоположными группами мышц, чтобы каждая мышца была на 100% в разгар тренировки.

    Время отдыха

    Время отдыха между тренировками чрезвычайно важно для достижения желаемых результатов. Рост мышц происходит не во время тренировки, а когда мышцы восстанавливаются и восстанавливаются во время отдыха.Если вы проработаете одну и ту же группу мышц до того, как она восстановится должным образом, вы действительно можете подавить ее рост. Хотя на время отдыха влияют многие факторы, Американский совет по упражнениям рекомендует дать мышечной группе отдохнуть два дня, прежде чем снова работать.

    Что такое сплиты тренировок и какой из них лучший?

    Когда вы заходите в тренажерный зал, вы представляете, что собираетесь делать, или вы находите первую пустую станцию ​​и начинаете откачивать количество повторений? Ради вас, мы надеемся, что это первое.

    Если ваша цель — нарастить мышечную массу и сжечь жир, стать сильнее или стать более спортивным — вам нужно знать, что вы делаете и почему вы это делаете. И важным шагом в составлении продуманного плана тренировок является выбор правильного разделения тренировок.

    Разделение тренировок — это то, как вы делите тренировки в течение недели по областям тела, движениям, конкретным частям тела или по подъемам. Такой подход к упражнениям, основанный на принципах «разделяй и властвуй», позволяет бодибилдерам и обычным посетителям тренажерного зала сосредоточить свои усилия на достижении оптимальных результатов.

    Вместо того, чтобы надеяться на более крупные мышцы или большую силу, вы гарантируете это, придерживаясь плана, который доказал свою эффективность. Разделение тренировок позволяет вам работать умнее и усерднее.

    В этой статье мы дадим вам всю информацию, необходимую для разработки собственного сплита тренировок, и несколько советов, как его придерживаться.

    Почему так важны разделение на тренировку?

    Разделы тренировок — это путь к определенной цели. Нет ни одного квотербека НФЛ, который не смотрел бы фильм о команде соперника или не разговаривал бы со своим О-образным перед выходом на решетку — это та же концепция для бодибилдеров и пауэрлифтеров.

    Content Studio / Shutterstock

    Но важность разделения выходит за рамки ясности ума — оно также дает результаты. Опрос 127 соревнующихся бодибилдеров показал, что каждый из них тренировался отдельно. (1)

    Это потому, что важность режима подчеркивалась с первых дней бодибилдинга. Стив Ривз и Юджин Сандоу разработали свои собственные программы для улучшения телосложения, которое по сей день многие считают вершиной бодибилдинга.

    Они изучили, как их тела реагируют на различные тренировки и время восстановления, нашли то, что им лучше всего подходит, и придерживались этого.

    Без плана ваши бесчисленные часы в тренажерном зале ни к чему не приведут. Развитие шпагата позволяет задействовать несколько мышц и истощить их. Затем вы даете им достаточно времени, чтобы прийти в себя и подготовиться к следующему занятию.

    Этот расчетный подход позволяет тренироваться с максимальным усилием. Вы не сгорите, пытаясь пройти трехчасовой марафон по прокачке железа.

    Как выбрать лучший?

    Первое, что нужно помнить при выборе сплита для тренировок, — это то, чего вы хотите достичь, но есть и другие вещи, о которых нужно помнить.

    Ваше время будет главным. Если у вас плотный рабочий график, шестидневный перерыв, вероятно, не будет вашим лучшим выбором.

    Ваш опыт обучения — еще один важный фактор. Допустим, вы настоящий новичок — вы, вероятно, захотите, чтобы ваши тренировки были более легкими, чтобы не перенапрягать мышцы.С другой стороны, опытному лифтеру, вероятно, понадобится больше стимулов для достижения желаемых результатов, поэтому он, скорее всего, будет искать что-то, что заставляет его заниматься в тренажерном зале больше дней в неделю.

    Наконец, вам будет на что обратить внимание на свои слабые стороны. Что мы имеем в виду? Если ваша слабость — недоразвитая середина, вам нужно сделать что-то более эстетичное, чем сила. Некоторые шпагаты подходят для этого лучше, чем другие.

    Как организованы сплиты на тренировке

    В конце концов, ваша тренировка будет зависеть от вашей конечной цели.Например, у тех, кто работает над улучшением своего атлетизма, будет другой раскол, чем у тех, кто сосредоточен исключительно на эстетике. То же самое и с пауэрлифтерами. Существует три основных режима тренировок, каждый из которых имеет свои преимущества и недостатки.

    Это:

    • Тренировка на части тела
    • Разделение тренировок на верх / низ
    • Тяга-толкание, сплит-тренировка ног

    Сплиты для тренировки частей тела

    Сплит-тренировка на части тела позволяет тренировать от одной до трех частей тела за тренировку два раза в неделю.Это популярный вариант среди бодибилдеров, поскольку разделенные части тела позволяют тренировать мышцы чаще для большего роста.

    Мета-анализ в журнале Sports Medicine показал, что этот стиль тренировок привел к наибольшей гипертрофии по сравнению с другими стилями тренировок. Это было подтверждено исследованием 2018 года, которое показало, что тренировка мышцы два раза в неделю приводит к увеличению толщины мышц и общего состава тела по сравнению с другими разделами тренировок. (2) (3)

    tsyhun / Shutterstock

    Основная цель бодибилдера — иметь полностью симметричное телосложение с полным развитием мышц.По этой причине большинство бодибилдеров объединяют более крупную мышцу, такую ​​как грудь, с более мелкой мышцей, такой как трицепс. Поскольку обе мышцы работают вместе в сложных упражнениях, таких как жим лежа и отжимания, имеет смысл объединить их вместе. Другие стандартные пары разделенных частей тела включают спину и бицепсы, а также ноги и плечи. Итак, разделение тренировок на части тела может выглядеть так.

    Пример тренировки сплита на части тела

    • Понедельник: Грудь и трицепсы
    • Вторник: Спина и бицепсы
    • Среда: Ноги и плечи
    • Четверг: Отдых
    • Пятница: Грудь и трицепс
    • Суббота: Спина и бицепсы
    • Воскресенье: Ноги и плечи

    Обычно мышцам для восстановления требуется около 48 часов отдыха.Изучите раскол выше, и вы увидите, что каждая группа мышц отдыхает три дня (или 72 часа). В некоторых случаях бодибилдер, который хочет выявить слабое место, может тренировать эту мышцу три раза в неделю, делая это менее напряженным днем.

    Например, предположим, что вы выполняете упражнение, описанное выше, но вам нужно усилить подколенные сухожилия. Сохраняйте шпагат как есть, но добавьте шесть наборов упражнений на подколенные сухожилия в конце первой тренировки груди и трицепсов. Шесть сетов — это не так уж много, чтобы вы не утомились, но умножьте шесть на 52 (количество недель в году), и вы получите 312 дополнительных подходов к работе на подколенные сухожилия.

    Плюсы сплит-тренировки на части тела

    • На протяжении всего занятия вы сосредотачиваетесь на двух мышцах.
    • Обеспечивает полное восстановление.
    • Во время тренировки требуется меньше оборудования.
    • Вы меньше устаете, поскольку не тренируете несколько мышц.

    Минусы сплита для тренировки частей тела

    • Если вы пропустите тренировку, вам понадобится больше времени, чтобы наверстать упущенное.
    • Вы можете с нетерпением ждать определенного сеанса.
    • Некоторые части тела могут восстанавливаться быстрее, чем другие.

    Верхний / нижний разделитель тренировок

    Разделение на верх / низ делит тренировки на дни для верхней и нижней части тела. Этот сплит отлично подходит для новичков, людей с плотным графиком и тех, кто стремится стать сильнее. Это заставляет лифтера уделять первоочередное внимание основам и сокращать жир в своей программе.

    При тренировке большего количества мышц за тренировку нужно подходить к упражнениям избирательно. Тренировка верхней части тела нацелена не только на грудь и трицепсы, но и на бицепсы, плечи и спину.Вместо четырех-пяти упражнений для груди вам нужно будет делать одно или два движения на каждую часть тела, иначе вы рискуете переутомиться и получить травму.

    Иван Кочергин / Shutterstock

    Вот почему мы рекомендуем сосредоточиться на сложных движениях, то есть упражнениях, которые задействуют более одной мышцы. Изоляционные (или односуставные) движения, такие как сгибания рук, разгибания груди и подъемы в стороны, должны в первую очередь выполняться на блоке для рубки.

    Нельзя сказать, что эти упражнения бесполезны, но они приносят меньше пользы.День для верхней части тела может включать жим лежа, тягу штанги, военный жим и подтягивания. Если вы тренируетесь достаточно усердно, у вас не останется много энергии для малых мышц.

    Одно из преимуществ разделения верха / низа состоит в том, что вы будете меньше заниматься в тренажерном зале. По сути, вы концентрируете свою рабочую нагрузку на четыре более коротких, хотя и более целенаправленных занятия в неделю. Не беспокойтесь об уровне своей силы. Вы все еще можете стать сильнее, если не сильнее, с меньшей частотой тренировок (то есть с частотой тренировок).

    На самом деле, исследование 2019 года показало, что разделение тренировок для верхних и нижних частей тела привело к увеличению размера мышц и увеличению силы по сравнению с общей тренировкой тела, выполняемой три раза в неделю. Как и при расщеплении частей тела, вы по-прежнему задействуете каждую мышцу на волшебной отметке два раза в неделю, которая считается оптимальной для бодибилдеров. (4)

    Однако бодибилдеры могут опасаться этого разделения, потому что в нем задействован меньший объем, то есть вы будете делать меньше повторений каждую неделю. На разделенной части тела больше места для вспомогательных подъемов — не так много на этой.

    Давайте возьмем пример подколенного сухожилия, который мы упоминали в предыдущем разделе. Это невозможно сделать на этом сплите, потому что вы все равно будете работать с ними на следующий день, поэтому вы не хотите их заранее изматывать. Та же идея применима к добавлению тренировок для верхней части тела в день для нижней части тела — вы просто истощили каждую мышцу в этой группе, поэтому вашему телу не захочется выполнить еще несколько подходов жима или тяги на следующий день.

    Пример разделения тренировки на верх / низ

    • Понедельник: Верхняя часть
    • Вторник: Нижняя часть тела
    • Среда: Отдых
    • Четверг: Верхняя часть
    • Пятница: Нижняя часть тела
    • Суббота и воскресенье: Отдых

    Плюсы сплита верхней / нижней части тренировки

    • Вы меньше ходите в спортзал.
    • Вы по-прежнему тренируете каждую мышцу дважды в неделю.
    • Можно работать над увеличением основных подъемников.

    Минусы верхней / нижней тренировки сплита

    • На тренировку меньше объема.

    Тяга, толчок, шпагат для ног

    Этот раздел тренировок аналогичен разделу «верхний / нижний». Основное отличие состоит в том, что PPL-сплит делит тренировку верхней части тела на две категории: тяга и толчок. Этот сплит преобладает в сообществе пауэрлифтеров, потому что они могут строить свою программу вокруг подъемов «большой тройки» — жима лежа (толчок), становой тяги (тяги) и приседаний (ноги).

    Он также обладает высокой масштабируемостью с точки зрения частоты. Занятые лифтеры могут интенсивно тренироваться (то есть с большим количеством упражнений для большего количества подходов и повторений) три раза в неделю. Люди, желающие чаще посещать тренажерный зал, могут снизить объем за тренировку и выполнять каждую тренировку два раза в неделю. Вы также можете тренироваться четыре раза в неделю и добавить дополнительный сеанс толкания, тяги или ног (в зависимости от того, над чем вам нужно работать).

    Jacob Lund / Shutterstock

    Если вы выберете вариант «шесть дней в неделю», подумайте об интенсивности тренировок и выборе упражнений.Во время первых трех занятий вы можете расставить приоритеты «большой тройки» и поднимать более тяжелые веса (это ваши силовые тренировки). Последние три занятия могут быть днями с большим объемом, чтобы укрепить более мелкие мышцы, такие как бицепсы, плечи и трицепсы (это ваши тренировки для гипертрофии).

    Шесть дней тренировок — это много, так что не переусердствуйте. Предположим, вы пауэрлифтер или силовой атлет. В этом случае вы можете использовать занятия с третьего по шестой, чтобы сосредоточиться на альтернативах «большой тройке», таких как присед на ящик, становая тяга с дефицитом и жим с пола.

    Тяга, толчок, тренировка ног, пример шпагата

    • Понедельник: Жим (тяжелый жим лежа)
    • Вторник: Тяга (тяжелая становая тяга)
    • Среда: Ноги (тяжелые приседания)
    • Четверг: Отдых
    • Пятница: Жим (альтернатива большому объему или жим лежа)
    • Суббота: Тяга (альтернатива большому объему или становая тяга)
    • Воскресенье: Ноги (альтернатива приседаниям с большим объемом или назад)

    Все ранее упомянутые исследования показали, что толкание / тяга / ноги являются жизнеспособным вариантом для роста мышц и увеличения силы.Вы по-прежнему задействуете каждую мышцу дважды в неделю и оставляете достаточно времени между тренировками, чтобы эти мышцы восстановились и подготовились к следующей тренировке.

    Плюсы толкания, тяги, ног

    • Упор на тренировку определенных мышц.
    • Достаточное время восстановления.

    Минусы толчка, толчка, ног

    • Меньше возможностей для модификации для устранения слабых мест.
    • Больше времени в спортзале.
    • Требуется дополнительное оборудование.

    Еще одна вещь — есть запасной план

    Составить план до того, как вы пойдете в спортзал, — это здорово, но что происходит, когда все идет не так, как вы хотите? Допустим, вы приходите в день ног, а люди возятся со стойками для приседаний.

    Скорее всего, у вас недостаточно времени, чтобы их дождаться — и именно тогда вам понадобится запасной план того, что вы запланировали на этот день. Если вы не можете приседать, попробуйте сделать тяжелые выпады или подъемы. Не можете найти открытый жим лежа? Замена жима гантелей. Вы уловили идею.

    Переполненные тренажерные залы — не единственное препятствие, которое нужно планировать. Скажем, метель — или глобальная пандемия (представьте?) — закрывает спортивные залы в вашем районе. Что вы делаете? Не тренируешься в тот день? Нет, вы найдете способ заставить его работать с тем, что есть в вашем распоряжении.

    Если у вас есть несколько лент сопротивления, используйте их. Нет оборудования? Существует бесчисленное количество упражнений с собственным весом, которые можно выполнять, чтобы проработать определенные мышцы.

    Предположим также, что вы путешествуете по работе. Не каждый тренажерный зал отеля укомплектован тренажерами Hammer Strength, а в некоторых может быть только несколько легких гантелей и беговые дорожки (у многих могут быть только последние). Опять же, заставьте его работать, адаптируя свои тренировки, либо изменив движения, которые вы запланировали на этот день, либо изменив схемы подходов и повторений.

    Еще лучше не забудьте взять с собой портативное тренировочное оборудование, если вы знаете, что можете находиться где-то с ограниченным оборудованием. Помимо эластичных лент, тренировочные системы с подвеской TRX — еще один легкий вариант, который легко помещается в чемодан и берет с собой куда угодно.

    Еще одна идея: поищите ближайший семейный тренажерный зал и посмотрите, дадут ли вам однодневный абонемент за небольшую плату. Если в тренажерном зале не слишком много людей, нет причин, по которым вас не могли бы позволить вам быстро потренироваться.

    Отсутствие одного дня здесь и там не убьет ваш прогресс, но разделение тренировок будет эффективным только в том случае, если вы действительно будете их придерживаться, поэтому иметь план действий в непредвиденных обстоятельствах необходимо, если вы хотите видеть результаты.

    Что нужно помнить

    • Определите конечную цель, прежде чем выбирать раскол.
    • Будьте реалистичны со своим временем.
    • Составьте план и придерживайтесь его.
    • Имейте запасной план.

    Список литературы

    1. Hackett DA, Johnson NA, Chow CM.Тренировочные практики и эргогенные средства, используемые бодибилдерами-мужчинами. J Strength Cond Res. 2013 июн; 27 (6): 1609-17. DOI: 10.1519 / JSC.0b013e318271272a. PMID: 229.
    2. Schoenfeld BJ, Ogborn D, Krieger JW. Влияние частоты тренировок с отягощениями на показатели мышечной гипертрофии: систематический обзор и метаанализ. Sports Med. 2016 ноя; 46 (11): 1689-1697. DOI: 10.1007 / s40279-016-0543-8. PMID: 27102172.
    3. Юэ Ф.Л., Карстен Б., Ларумбе-Забала Э., Сейджо М., Наклерио Ф. Сравнение двух еженедельных тренировок с отягощениями с выравниванием объема, использующих различные частоты состава тела и производительности у тренированных мужчин.Appl Physiol Nutr Metab. 2018 Май; 43 (5): 475-481. DOI: 10.1139 / apnm-2017-0575. Epub 2017, 7 декабря. PMID: 29216446.
    4. Lasevicius T, Schoenfeld BJ, Grgic J, Laurentino G, Tavares LD, Tricoli V. Подобные мышечные адаптации при тренировках с отягощениями выполнялись два или три дня в неделю. Джум Кинет . 2019; 68: 135-143. Опубликовано 21 августа 2019 г. doi: 10.2478 / hukin-2019-0062

    Featured image: Jacob Lund / Shutterstock

    Как накачать мышцы, чтобы лучше выглядеть в футболке

    Очевидно, что атака на большие группы мышц верхней части тела — спину, грудь и плечи — важна, но ключ кроется в мельчайших деталях.Другими словами, все дело в том, чтобы маленькие мышцы внутри больших мышц выглядели лучше в вашей футболке. Например, ваш бицепс состоит из двух частей — двуглавой мышцы плеча в верхней части плеча и плечевой мышцы, которая начинается немного ниже и помогает вам согнуть локоть. Многие люди сосредотачиваются только на первом, но нацеливание на меньший компонент двуглавой мышцы — плечевую мышцу — добавляет дополнительный уровень детализации. (Не волнуйтесь, мы зажжем их молоточковыми локонами на тренировке ниже.)

    На самом деле трицепс занимает больший процент вашей руки, чем бицепс. Они состоят из трех частей (отсюда и название): боковой, медиальной и длинной головок. Это означает, что вы хотите увеличить объем бицепса немного выше, чем если бы вы это делали. Вот почему мы увеличили количество подходов для отжиманий и разжиманий черепа с гантелями в Тренировке Б.

    И когда вы тренируете плечи, это не может быть только прессинг (который нацелен на ваши передние и средние дельты).Многие парни пренебрегают развитием задних дельт, но вам нужно найти способы проработать их, чтобы придать себе трехмерную симметрию. Вы добьетесь этого с помощью упражнений на верхнюю часть спины, что для многих станет неожиданностью. Вот почему мы включили задние дельты в тренировку Б.

    .

    Говоря о вашей спине, тренировки не могут сводиться только к становой тяге и гребле. Вам нужны упражнения, которые подчеркнут V-образный конус, который поможет заполнить фигуру. Если вы гребец, возможно, вы думаете, что вам не нужно подтягивание широким хватом, но вам это нужно.Гребля работает на ваши трапеции, но подтягивания широким хватом также в большей степени задействуют латимус спины и ромбовидные мышцы гораздо меньшего размера.

    И чтобы закончить работу над мускулами футболки, у нас есть день груди. Убедитесь, что вы выполняете упражнения, которые воздействуют на мышечные волокна, составляющие ключичную часть верхней части груди, чтобы заполнить линию декольте и создать иллюзию большей груди. И хотя большая грудная мышца звучит более важно, удары по малой грудной мышце движениями, такими как отжимания, не менее важны.

    Мы просто бросили вам много физиологии, но вам не нужно ничего из этого запоминать, потому что мы выполнили сложную часть и разработали две тренировки, которые учитывают все это — помогая вам лепить не только эти большие, основные мышцы, но также и более мелкие, которыми иногда пренебрегают, так что вы будете лучше выглядеть в футболке.

    Инструкции: Выполняйте тренировку A и B один раз в неделю с как минимум одним выходным днем ​​между ними (в этот день вы можете сосредоточиться на нижней части тела).

    Тренировка любезно предоставлена ​​Алиссой Эйджес, спортсменкой с PowerNYC Training и тренером в Uplift Studios, EPIC Hybrid Training и Global Strongman Gym.

    Как накачать мышцы, чтобы лучше выглядеть в футболке

    Тренировка A

    Основные задействованные мышцы: широчайшие, бицепсы, грудь

    1. Подтягивания на тросе — 4 x 10 повторений
    Как это делать: Сядьте на тренажер с тросом для широчайшего, вытяните руки широким хватом и потяните прямую штангу вниз до уровня чуть ниже подбородка (это должно быть трудно тянуть штангу дальше). Медленно вернитесь к началу и повторите

    2.Подтягивания широким хватом — 4 x AMRAP (как можно больше повторений)
    Как это делать: Для наращивания широчайших мышц выполняйте подтягивания более широким хватом, примерно вдвое шире плеч.

    3. Сгибания рук с молотка — 4 x 8 повторений
    Как это делать: Держите гантели в каждой руке ладонями друг к другу. Держа предплечья по бокам, сгибайте оба веса одновременно.

    4. Концентрированные сгибания рук сидя — 4 x 10 повторений на каждую сторону
    Как это делать: Сядьте на край скамьи с гантелью в одной руке и расставьте ступни на ширине плеч.Положите руку с гантелью на внутреннюю поверхность бедра. Для начала согните гантель к плечу и пояснице.

    5. Пуловер с гантелями лежа — 4 x 10 повторений
    Как это делать: На скамье или на полу жмите две гантели вместе, руки прямо над грудью. Держа руки прямыми, медленно поднимите гантели над головой к полу или скамейке. Вернитесь в центр.

    6. Жим штанги на наклонной скамье — 4 x 6-8 повторений
    Как это делать: Установите наклонную скамью под углом примерно 45 градусов.Опустите штангу к верхней части груди и вернитесь наверх.

    Тренировка B

    Основные задействованные мышцы: трапеции, трицепсы, плечи

    1. Шраги со штангой — 4 x 8 повторений
    Как это делать: Держите штангу перед собой ногами на ширине плеч. Ладони должны быть обращены к вашему телу. Поднимите плечи, задержитесь и вернитесь в исходное положение.

    2. Гиря сумо High Pull — 4 x 10 повторений
    Как это делать: Встаньте, ноги в широкую стойку, носки слегка разведены.Поместите гирю между ног и присядьте, чтобы захватить ее обеими руками. Когда вы пытаетесь встать через пятки, подтяните гирю на высоту плеч, приподняв локти. Вернуться к началу.

    3. Отжимания — 5 x AMRAP
    Как это делать: Используя параллельные перекладины, опустите тело до тех пор, пока ваши руки не будут под углом 90 градусов, и отожмите назад до прямых рук.

    4. Раздавливание черепа с гантелями — 5 x 10-12 повторений
    Как это сделать: Лежа на спине на полу, держите гантели прямо над грудью. .Повернувшись в локтях, опустите гантели к голове так, чтобы они касались пола по обе стороны от головы.

    5. Строгий жим штанги над головой — 4 x 6-8 повторений
    Как это делать: Держа штангу на высоте плеч в стойке для приседаний, возьмитесь за штангу шире, чем ширина плеч. Выйди из стойки. Не опускаясь, жмите штангу над головой, сохраняя вертикальную траекторию, пока штанга полностью не окажется над головой. Соедините руки и медленно вернитесь в исходное положение у плеч.

    6. Подъемы гантелей в стороны — 3 x 10-12 повторений
    Как это делать: Стоя, ноги на ширине плеч, держите по одной гантели в каждой руке. Поднимите руки в стороны от тела, пока они не окажутся на уровне плеч. Медленно вернитесь к началу.

    7. Задняя дельта-муха — 3 x 10-12 повторений
    Как это делать: Поверните вперед от бедер, держа по гантели в каждой руке, сохраняя небольшой сгиб в коленях. Поднимите гантели по бокам тела ладонями вниз, пока они не сравняются с вашими плечами.Медленно поверните движение в обратном направлении, чтобы вернуться к началу.

    Бонусные движения для наращивания мышц на футболке

    Эти упражнения прорабатывают несколько групп мышц верхней части тела и могут быть добавлены в любой день, чтобы действительно накачать мышцы и выглядеть лучше в футболке.

    1. Становая тяга со штангой — 4 x 6 повторений
    Как это делать: Встаньте перед штангой и согнитесь в коленях, сохраняя ровную спину, чтобы захватить штангу сверху или смешанным (если вес Вам сложно двигаться) сцепление.Напрягите верхнюю часть спины и прямыми руками стяните со штанги слабину. Отжимая пятки, встаньте со штангой и зафиксируйте ее, чтобы завершить движение. Вернуться к началу.

    2. Manmakers — 2 x 10 повторений
    Как это сделать: Любимый кроссфиттер, этот тренирует ряд мышц верхней части тела. Начните с положения планки с гантелями под каждой рукой. Выполните одно отжимание. Выполните ренегатский ряд каждой рукой. Поднимите ноги к рукам и присядьте с гантелями на плечах.Когда вы стоите, используйте привод ног, чтобы жать гантели над головой.

    Чтобы получить доступ к эксклюзивным видео о снаряжении, интервью со знаменитостями и многому другому, подпишитесь на YouTube!

    Какие мышцы нельзя прорабатывать в один день | Живите здоровым

    Мари Малруни Обновлено 29 апреля 2019 г.

    В определенный день вы можете объединить любые группы мышц в тренировку — при условии, что эти мышцы здоровы и вы дадите им достаточно времени для восстановления, прежде чем снова их проработать.Для бодибилдеров или тех, кому нужно дополнительное время, чтобы сосредоточиться на определенной группе мышц, сплит-тренировки, которые в определенные дни прорабатывают только определенные мышцы, могут быть эффективным инструментом.

    Фокус на все тело

    Хотя бодибилдинг сплиты — проработка только определенных групп мышц при каждом подъеме — довольно популярны, тренировки всего тела имеют ряд преимуществ для новичков. Во-первых, их легче вписать в плотный график, потому что вы можете быстро входить в тренажерный зал и выходить из него, не экономя на тренировке, чтобы сэкономить время.Во-вторых, это быстрый и эффективный способ развития общей силы и выносливости, которые необходимы большинству новичков для начала. И в-третьих, выполнение тренировок для всего тела означает, что вам не нужно осваивать столько различных упражнений, сколько вам нужно для выполнения сплит-тренировок.

    Рассмотрите сплит

    Сплит-тренировки позволяют вам уделять дополнительное время определенной группе мышц или технике; например, некоторые бодибилдеры проводят целую тренировку, прорабатывая определенную группу мышц со всех возможных углов. Разделенные тренировки дают вам время, необходимое для интенсивного сосредоточения на этой группе мышц или технике, а затем дают этим мышцам достаточно времени для восстановления, поскольку вы прорабатываете другой набор мышц в последующие дни.Менее интенсивные тренировки также могут быть полезны, если у вас действительно мало времени; вы можете втиснуть пару упражнений для груди и спины в один день, упражнения для ног на следующий день, а затем упражнения для рук на третий день.

    Следуйте стандартным сплитам

    То, как вы «разбиваете» группы мышц между тренировками, в значительной степени определяется тем, сколько дней в неделю вы планируете проводить упражнения. Например, типичный двухдневный сплит прорабатывает нижнюю часть тела в один день и верхнюю часть тела на следующий день.